oite_ mcq´s questionsfinal2011

67
2011 Orthopaedic In-Training Examination • 1 1 Tibial tubercle osteotomy has proven to be an effective treatment in the management of articular cartilage injuries of the patella. It is contraindicated for chondral injuries in what location on the patella? 1- Lateral 2- Distal 3- Proximal 4- Central 5- Medial 2 For potential injury to the S2 nerve root following a sacral fracture, the physician should test for 1- perianal sensation. 2- great toe dorsiflexion. 3- ankle toe plantarflexion. 4- sensation on the medial border of the foot. 5- sensation in the first web space of the foot. 3 A 16-year-old girl has a significant valgus deformity of her right leg. Examination of the lower limb deformity reveals an anatomic lateral distal femoral angle of 61 degrees, an anatomic medial proximal tibial angle of 88 degrees, and a tibial femoral angle of 26 degrees. All other measurements are within normal limits. What is the most appropriate treatment? 1- Distal femoral varus osteotomy 2- Distal femoral valgus osteotomy 3- Proximal tibial varus osteotomy 4- Proximal tibial valgus osteotomy 5- Combined distal femoral varus and proximal tibial valgus osteotomies 4 Figure 4 are the radiographs of a 13-year-old boy who sustained a forearm injury 3 days ago. Management should consist of 1- percutaneous skeletal fixation. 2- open reduction and internal fixation. 3- closed reduction and short-arm cast immobilization. 4- closed reduction and long-arm cast immobilization in supination. 5- closed reduction and long-arm cast immobilization in pronation.

Upload: katkut99

Post on 28-Oct-2015

161 views

Category:

Documents


24 download

TRANSCRIPT

2011 Orthopaedic In-Training Examination • 1

1 Tibial tubercle osteotomy has proven to be an effective treatment in the management of articular cartilage injuries of the patella. It is contraindicated for chondral injuries in what location on the patella? 1- Lateral 2- Distal 3- Proximal 4- Central 5- Medial 2 For potential injury to the S2 nerve root following a sacral fracture, the physician should test for 1- perianal sensation. 2- great toe dorsiflexion. 3- ankle toe plantarflexion. 4- sensation on the medial border of the foot. 5- sensation in the first web space of the foot. 3 A 16-year-old girl has a significant valgus deformity of her right leg. Examination of the lower limb deformity reveals an anatomic lateral distal femoral angle of 61 degrees, an anatomic medial proximal tibial angle of 88 degrees, and a tibial femoral angle of 26 degrees. All other measurements are within normal limits. What is the most appropriate treatment? 1- Distal femoral varus osteotomy 2- Distal femoral valgus osteotomy 3- Proximal tibial varus osteotomy 4- Proximal tibial valgus osteotomy 5- Combined distal femoral varus and proximal tibial valgus osteotomies 4 Figure 4 are the radiographs of a 13-year-old boy who sustained a forearm injury 3 days ago. Management should consist of 1- percutaneous skeletal fixation. 2- open reduction and internal fixation. 3- closed reduction and short-arm cast immobilization. 4- closed reduction and long-arm cast immobilization in supination. 5- closed reduction and long-arm cast immobilization in pronation.

American Academy of Orthopaedic Surgeons • 2

5 For the purposes of retractor and screw placement, the anterior and posterior zones of the acetabulum are divided by a line 1- directly along the body axis. 2- between the posterior superior iliac spine and the obturator foramen. 3- between the anterior superior iliac spine and the ischial tuberosity. 4- between the anterior superior iliac spine and the center of the acetabulum. 5- from the center of the acetabulum to the transverse acetabular ligament. 6 The anterolateral approach to the distal tibia and ankle for open reduction and internal fixation of pilon fractures places which of the following nerves at most risk? 1- Sural 2- Saphenous 3- Deep peroneal 4- Medial plantar 5- Lateral plantar 7 Assuming that all pins and bars are of equivalent size and material, and the bone is loaded axially at each end, which of the following external fixation constructs seen in Figures 7a through 7e will produce the least motion between bone ends? 1- Figure 7a 2- Figure 7b 3- Figure 7c 4- Figure 7d 5- Figure 7e 8 Figure 8 shows the clinical photograph of a 3-month-old girl who has a circumferential crease around her arm. She is neurovascularly intact. The hand appears to function normally, but she has significant swelling of the forearm when it is dependent. Treatment should consist of 1- above-elbow amputation. 2- through-elbow amputation. 3- resection of the redundant skin. 4- circumferential band excision and z-plasty. 5- radical skin resection and reconstruction at age 18 months.\

2011 Orthopaedic In-Training Examination • 3

9 A man has a T5 burst fracture. Examination reveals 0/5 motor strength in both lower extremities in all groups. He has no light touch or pinprick sensation in his lower extremities or trunk. He has decreased rectal tone but has intact perianal sensation and an intact bulbocavernosus reflex. According to the American Spinal Injury Association (ASIA) grading scale, what is the classification of spinal cord injury? 1- ASIA A, complete 2- ASIA A, incomplete 3- ASIA B, complete 4- ASIA B, incomplete 5- ASIA C, incomplete 10 A 78-year-old man has long-standing ankle stiffness and pain that is aggravated by walking. Corticosteroid injection has not relieved his pain. He has a painful 10-degree arc of ankle motion. Radiographs reveal end-stage degenerative arthritis. Which of the following devices will be most effective? 1- Heel lift 2- Single rocker sole shoe modification 3- Double rocker sole shoe modification 4- Medial heel flare shoe modification 5- Lateral heel flare shoe modification 11 Secondary osteosarcomas have been observed most frequently in which of the following conditions? 1- Melorheostosis 2- Paget's disease 3- Cortical desmoids 4- Fibrous dysplasia 5- Osteoporosis in patients treated with bisphosphonates 12 Femoral nailing through the piriformis fossa starting portal is contraindicated in adolescents with open physes because of the risk of 1- injury to the sciatic nerve. 2- injury to the greater trochanteric apophysis. 3- injury to the medial femoral circumflex artery. 4- injury to the lateral femoral circumflex artery. 5- increased intramedullary pressure.

American Academy of Orthopaedic Surgeons • 4

13 Which of the following patients would be the best candidate for surgical "guided growth technique" using a tension band plate(s) across the medial distal femoral growth plate(s)? 1- Bone age 5-year-old girl with bilateral genu varum with wide irregular growth plates 2- Bone age 11-year-old girl with bilateral rachitic genu varum 3- Bone age 11-year-old girl with bilateral severe genu valgum 4- Bone age 12-year-old boy with unilateral posttraumatic genu valgum and a lateral femoral growth arrest 5- Bone age 12-year-old boy with unilateral posttraumatic genu varum and a medial femoral growth arrest 14 Figures 14a through 14d are the radiographs of a 55-year-old man who sustained a dislocation. He undergoes immediate closed reduction and is seen 3 days later. His skin is intact and he is able to contract his deltoid. Management should now consist of 1- hemiarthroplasty. 2- immediate passive therapy. 3- sling immobilization for 6 weeks. 4- open reduction and internal fixation. 5- external rotation immobilization for 2 weeks. 15 Neuroblastoma is the most common metastatic malignant solid tumor in childhood and arises from which of the following? 1- Eosinophils 2- Histiocytes 3- Macrophages 4- Nucleus pulposus cells 5- Neural-crest derived cells 16 A 55-year-old man who is an avid golfer undergoes a total hip arthroplasty. What effect would this procedure have on his golf game? 1- A likely return to play with a minimal change in handicap 2- Return to play is not recommended because of the risk of premature wear 3- Return to play with a significant increase in average drive distance 4- No need to use a golf cart following hip arthroplasty 5- More pain during the golf swing associated with left-sided surgery compared with the right side

2011 Orthopaedic In-Training Examination • 5

17 A 56-year-old woman sustained a displaced ankle fracture and underwent surgical fixation 4 weeks ago. She is now seen for postoperative assessment, and current radiographs are seen in Figures 17a and 17b. Tibiotalar relationships are most likely the result of 1- fibular malreduction. 2- inadequate syndesmotic screw length. 3- inadequate syndesmotic screw diameter. 4- medial collateral ligament incompetence. 5- entrapment of the flexor hallucis longus tendon. 18 Figures 18a through 18c are the radiographs of a 29-year-old man involved in a snowmobile accident. He sustained an open book pelvic injury, midshaft femur fracture, and an undisplaced thoracic spine fracture. The pelvis and femur fractures required surgical internal fixation and the spine fracture is being treated with a brace. Management of these upper extremity injuries should consist of 1- above-elbow casting of the forearm and hand. 2- clamshell splinting of the ulnar shaft and pinning of the finger fractures. 3- clamshell splinting of the ulnar shaft fracture and thermoplast splinting of the fingers. 4- open plating of the ulnar shaft and metacarpals, and fixation of the proximal phalanx. 5- open plating of the ulnar shaft and metacarpals, and closed treatment of the proximal phalanx. 19 Figure 19 is a three-dimensional CT reconstruction of the shoulder of a 22-year-old man following a shoulder dislocation. What is the most appropriate management? 1- Open stabilization and shortening of the subscapularis 2- Open stabilization to address the articular arc deficit from the osseous Bankart lesion 3- Sling immobilization followed by progressive range of motion and strengthening 4- Arthroscopic stabilization with tenodesis of the subscapularis to the defect 5- Arthroscopic stabilization with removal of the fragment and reattachment of the capsulolabral complex 20 A 55-year-old woman has metastatic breast cancer. She is given a nitrogen-containing bisphosphonate to inhibit bone absorption. What is the mechanism of action for this therapy? 1- Disruption of isoprenylation 2- Reduction of carbonic anhydrase activity 3- Tumor inhibition due to tumor cell apoptosis 4- Interference with osteoclast adhesion 5- Interference of tumor cells ability to absorb bone

American Academy of Orthopaedic Surgeons • 6

21 A 3-year-old boy has a septic hip. You suspect that Kingella kingae is the causative organism. What culture medium should be requested? 1- Agar plates 2- Thayer-Martin plates 3- Blood culture medium 4- Egg-based mediums 5- Luria Bertani medium 22 Which of the following is considered the preferred method of predicting hip stability after a posterior wall acetabular fracture? 1- Dynamic fluoroscopic examination under anesthesia 2- The Moed method (Figure 22a) 3- The Calkins method (Figure 22b) 4- The Keith method (Figure 22c) 5- A history of associated hip dislocation 23 The muscle contraction characterized by constant muscle tension through the range of motion is called 1- isotonic. 2- isometric. 3- isoelastic. 4- isokinetic. 5- isoconcentric. 24 What is the most common complication of the Weil distal metatarsal osteotomy when combined with a proximal interphalangeal joint resection arthroplasty? 1- Floating-toe deformity 2- Nonunion of the metatarsal osteotomy 3- Recurrent proximal interphalangeal joint deformity 4- Recurrent dislocation of the metatarsophalangeal joint 5- Intractable plantar keratosis at the metatarsophalangeal joint

2011 Orthopaedic In-Training Examination • 7

25 Which of the following has been shown to be a risk factor for pseudarthrosis following long adult spinal deformity surgery? 1- Thoracoabdominal approach 2- Kyphosis of less than 20 degrees 3- Preexisting knee osteoarthritis 4- Positive sagittal balance of less than 2 cm 5- Instrumentation of the upper thoracic spine 26 A 60-year-old woman reports the sudden onset of severe hip pain after undergoing total hip arthroplasty 6 weeks ago. Radiographs reveal a dislocation. Immediate postoperative and postreduction radiographs are seen in Figures 26a through 26c. What is the most likely cause of the dislocation? 1- Cup position 2- Excess anteversion 3- Lack of femoral offset 4- Subsidence of the implant 5- Failure to adhere to hip precautions 27 When comparing outcomes between hemiarthroplasty and total shoulder arthroplasty for patients with symptomatic osteoarthritis with an intact rotator cuff and a concentric joint, hemiarthroplasty results in which of the following? 1- Equivalent pain relief 2- Increased blood loss 3- Increased rate of revision 4- Increased rate of instability 5- Increased incidence of subscapularis insufficiency 28 A 42-year-old man undergoes direct repair of an acute patellar tendon rupture. To permit early knee range of motion while protecting the integrity of the repair, which of the following methods is recommended? 1- Standing squats 2- Closed chain kinetic quadriceps exercises 3- Active knee flexion in a prone position 4- Active range of motion in a seated position without resistance 5- Active-assisted range of motion in a seated position without resistance

American Academy of Orthopaedic Surgeons • 8

29 In addition to an appropriate course of antibiotics, which of the following is the most appropriate definitive treatment for a child who has acute hematogenous osteomyelitis and a 2-cm by 2-cm abscess within the distal femur metaphysis? 1- Percutaneous biopsy and culture 2- Percutaneous biopsy, culture, curettage 3- Percutaneous biopsy, culture, curettage, and bone grafting 4- Open biopsy, culture, and debridement 5- Open biopsy, culture, and en bloc resection of the distal femur with application of external fixation 30 Which of the following fluoroscopic views is required for the proper introduction of iliosacral screws within the pelvis? 1- Anteroposterior 2- Iliac oblique 3- Obturator oblique 4- Obturator inlet 5- Sacral lateral 31 In Figure 31, an iliosacral screw placed into the first sacral segment in the trajectory shown by the white line places what structure at risk? 1- L4 nerve root 2- L5 nerve root 3- S1 nerve root 4- Superior gluteal nerve 5- Inferior gluteal nerve 32 Figures 32a through 32d are the radiographs and MRI scans of a 19-year-old woman with a 4-week history of right thigh pain. A biopsy specimen is seen in Figure 32e. What is the most likely diagnosis? 1- Osteomyelitis 2- Ewing's sarcoma 3- Multiple myeloma 4- Metastatic carcinoma 5- Eosinophilic granuloma

2011 Orthopaedic In-Training Examination • 9

33 A 56-year-old woman sustained a left distal radius fracture when she fell getting out of a chair. In addition to fracture care, the next step in assessment and management should include which of the following? 1- A bone scan 2- Pelvis and spine radiographs 3- Ultrasound bone density testing 4- Dual-energy x-ray absorptiometry 5- Urinary pyridinolines and N-telopeptides 34 Lateral retinacular release is most indicated in the treatment of 1- patella alta. 2- patellar subluxation. 3- patellofemoral instability. 4- symptomatic medial synovial plica. 5- lateral facet compression syndrome. 35 The highest rate of associated nerve injury in sacral fractures occurs in 1- Denis 1, which is a fracture lateral to the foramen. 2- Denis 1, which is a fracture medial to the foramen. 3- Denis 2, which is a fracture through the foramen. 4- Denis 3, which is a fracture lateral to the foramen. 5- Denis 3, which is a fracture medial to the foramen. 36 Approximately how many milligrams of calcium are in an 8-ounce cup of milk? 1- 75 2- 150 3- 300 4- 600 5- 1,200 37 Figures 37a through 37e are the radiographs and MRI scans of a 67-year-old woman with pain radiating into both legs with ambulation. Her pain is improved with bending forward or lying flat. Neurologic examination reveals 4/5 strength in ankle dorsiflexion but is otherwise normal. Nonsurgical management for the past 4 months has failed to provide relief, and she reports progressively worsening pain. What is the most effective management?

American Academy of Orthopaedic Surgeons • 10

1- Lumbar orthosis 2- Epidural steroid injection 3- Posterior L4-5 decompression 4- Posterior L4-5 uninstrumented arthrodesis 5- Posterior L4-5 lumbar decompression and arthrodesis 38 A nerve palsy involving what nerve is most common with halo cervical traction? 1- Cranial nerve V 2- Cranial nerve VI 3- Cranial nerve VII 4- Cranial nerve X 5- Cranial nerve XII 39 A 75-year-old man has pain in the proximal thigh after a fall. He appears to be in good health, and reported mild thigh pain prior to his fall. The radiograph reveals a transverse fracture of the proximal shaft of the femur with some lateral cortical thickening at the fracture site. What aspect of the patient's history most likely explains this fracture? 1- Smoking 2- IV drug use 3- Long-term steroid use 4- Chronic bisphosphonate use 5- Family history of rickets 40 Figure 40 is the radiograph of a 55-year-old man who has pain and swelling along the lateral aspect of the foot after stepping off a curb. He denies any pain in the region prior to the injury. Treatment should consist of 1- placement of an intramedullary screw. 2- application of a bone growth stimulator. 3- short-leg non-weight-bearing cast for 6 weeks. 4- walker boot for 6 weeks with weight bearing as tolerated. 5- postoperative shoe with weight bearing as tolerated until the pain subsides. 41 A 35-year-old man with a pilon fracture is ambulating with crutches. He moves both crutches and the injured limb forward bearing his weight on the crutches and then follows with all of his weight on the uninjured limb. He then repeats the pattern. This pattern of ambulation with an assistive device is described as a 1- 2-point gait.

2011 Orthopaedic In-Training Examination • 11

2- 3-point gait. 3- 4-point gait. 4- steppage gait. 5- swing through gait. 42 You have evaluated a new serologic test on a group of patients with a known disorder and a control group who does not have it. Based on the results seen in Figure 42, what is the sensitivity of the assay? 1- 70% 2- 75% 3- 80% 4- 88% 5- 90% 43 The surgical treatment of a 9-year-old child with displaced diaphyseal femur fractures with elastic intramedullary nails has been shown to have an increased rate of malunion in which of the following situations? 1- Use of stainless steel nails 2- Weight of greater than 110 pounds 3- Treatment delayed beyond 12 hours 4- Failure to use a spica cast postoperatively 5- Failure to lock the implants proximally or distally 44 What type of femoral fracture is best suited for locked plate fixation? 1- Lateral femoral condyle 2- Comminuted diaphyseal 3- Comminuted metaphyseal 4- Short oblique diaphyseal 5- Short oblique metaphyseal 45 Which of the following findings is most associated with intimate partner violence? 1- Lower extremity fracture 2- Multiple extremity fractures 3- Isolated abdominal injury 4- Evidence of drug or alcohol use 5- Pattern of repeated visits to the physician's office or the emergency department

American Academy of Orthopaedic Surgeons • 12

46 A 60-year-old woman underwent total hip arthroplasty 7 years ago and now has persistent groin pain. A bone scan and radiograph are seen in Figures 46a and 46b. Laboratory studies reveal a C-reactive protein of 20 mg/dL (normal < 2.0 mg/dL) and an erythrocyte sedimentation rate of 50 mm/h (normal up to 20 mm/h). Hip aspiration shows white blood cell count of 40,000/mm3 with 80% polynuclear cells. Cultures are negative. What is the most appropriate management? 1- Physical therapy 2- Revise acetabulum only 3- One-stage revision - all components 4- Two-stage revision - all components 5- Irrigation and debridement with polyethylene exchange 47 What type of tissue has the greatest blocked expression by selective Cox-2 inhibitors? 1- Platelet 2- Periosteum 3- Vascular endothelium 4- Tracheal endothelium 5- Gastrointestinal epithelium 48 What characteristic(s) do calcium sulfate and calcium phosphate cements share in common? 1- Both have similar porosity 2- Both have similar resorption rate 3- Both have similar compressive strength 4- Both have osteoinductive properties 5- Both resist tension and shear stresses poorly 49 Figure 49 is the standing PA radiograph of an otherwise healthy 14-year-old boy who failed scoliosis screening. He is neurologically intact and does not have back pain. Further evaluation should include which of the following? 1- Renal ultrasound 2- Echocardiography 3- Limb-length evaluation 4- Ocular slit lamp evaluation 5- MRI scan of the entire spine 50 Figure 50 is the clinical photograph of a 69-year-old man who underwent shoulder hemiarthroplasty 3 years ago with excellent pain relief. He currently has minimal pain, no fevers, and normal inflammatory

2011 Orthopaedic In-Training Examination • 13

markers, but has chronic drainage from the wound and loss of forward elevation. Active forward elevation is 30 degrees and there is no active external rotation. Aspiration 3-day cultures are negative for any aerobic or anaerobic organisms. Radiographs show a well-fixed component with no evidence of complication. What is the most appropriate management? 1- Latissimus dorsi tendon transfer 2- One-stage revision to total shoulder arthroplasty 3- One-stage revision to reverse total shoulder arthroplasty 4- Irrigation and debridement with tissue culture held for 2 weeks 5- No further treatment because cultures and laboratory studies are negative 51 Osteoclasts have specific receptors to ligands that allow them to regulate bone resorption, including 1- estrogen. 2- calcitonin. 3- 1,25 vitamin D3. 4- PTH (parathyroid hormone). 5- TNF (tumor necrosis factor) alpha. 52 Figures 52a and 52b are the radiographs of a right-hand dominant 17-year-old girl with wrist pain that began insidiously 3 months ago. It is aggravated by writing. There is an audible clunk when her wrist is passively moved from radial to ulnar deviation under axial load. She is ligamentously lax. What is the most likely diagnosis? 1- Dorsal wrist ganglion 2- Mid-carpal instability 3- Osteoid osteoma of the hamate 4- Scapholunate interosseous ligament tear 5- Lunotriquetral interosseous ligament tear 53 What tract is the main descending motor pathway in the cervical spinal cord? 1- Posterior column 2- Lateral corticospinal 3- Anterior corticospinal 4- Lateral spinothalamic 5- Anterior spinothalamic 54 What is the most important predictor of infection after an open type III tibial fracture? 1- Time to transfer to definitive trauma center

American Academy of Orthopaedic Surgeons • 14

2- Having an antiseptic dressing placed in the field 3- Fracture stabilization within 6 hours 4- Getting to the operating room within 6 hours 5- Receiving broad-spectrum antibiotics within 12 hours 55 Figures 55a and 55b are the MRI scans of a healthy 65-year-old man with a 3-month history of difficulty with his gait. No injury is reported. He describes unilateral ankle weakness and difficulty with stair climbing. Management should consist of which of the following? 1- Primary surgical repair 2- MRI of the lumbar spine 3- Ankle arthroscopy and debridement 4- Cast immobilization for 4 weeks to 6 weeks 5- Surgical reconstruction with tendon transfer/interposition 56 In a 2-year-old child, what neurovascular structure is most likely to be injured while performing a trigger thumb release? 1- Princeps pollicis artery 2- Ulnar digital nerve 3- Radial digital nerve 4- Thenar motor branch of the median nerve 5- Palmar cutaneous branch of the median nerve 57 A 31-year-old woman with end-stage renal disease on hemodialysis has severe, sudden onset left hip pain with no antecedent trauma. Figure 57a shows a current AP radiograph of the pelvis and Figure 57b shows her radiograph 2 months prior. A CT scan of the pelvis was also performed approximately 1 month prior with representative axial cuts through the sacroiliac joints shown in Figure 57c. Microphotographs of biopsy specimens are shown in Figures 57d and 57e. What is the most likely diagnosis? 1- Hypogonadism 2- Multiple myeloma 3- Giant cell tumor 4- Hyperparathyroidism 5- Renal cell carcinoma 58 Which of the following conditions is associated with renal osteodystrophy? 1- Hypoparathyroidism 2- Hyperparathyroidism

2011 Orthopaedic In-Training Examination • 15

3- Hypothyroidism 4- Hyperthyroidism 5- 25 (OH) vitamin D2 deficiency 59 To perform dynamic locking of the femoral nail seen in Figure 59, locking screw(s) should be placed in what position(s)? 1- A only 2- B only 3- C only 4- A and B 5- A and C 60 A 56-year-old man reports persistent, progressively worsening pain after undergoing total hip arthroplasty 18 months ago. A postoperative radiograph is shown in Figure 60a and a current AP hip radiograph is shown in Figure 60b. Based on the radiographic finding, what is the most likely diagnosis? 1- Infection 2- Osteolysis 3- Metal sensitivity 4- Aseptic loosening 5- Periprosthetic fracture 61 Which of the following is the most common neurologic complication following surgical reduction of a high-grade spondylolisthesis at L5-S1 in a pediatric patient? 1- L4 nerve root injury 2- L5 nerve root injury 3- S1 nerve root injury 4- S2 nerve root injury 5- Cauda equina syndrome 62 In which of the following scenarios is intraosseous infusion considered most appropriate? 1- Adult in a hypovolemic state 2- Required infusion rate in excess of 40 mL/min 3- Required rapid delivery of blood products and antibiotics 4- Previous attempted access at same site within 24 hours 5- Normotensive child older than 8 years with poor peripheral IV access

American Academy of Orthopaedic Surgeons • 16

63 Figures 63a and 63b are the radiographs of a 40-year-old male soldier with a painful middle fingertip. He sustained a fracture 9 months ago and management consisted of a splint for the first 2 months. He has a significant nail deformity and is unable to grip with the fingertip because of pain. Management should now consist of 1- reduction and percutaneous wire fixation. 2- spiral oblique retinacular ligament reconstruction. 3- continued full-time splinting until the fracture heals. 4- application of a pulsed electromagnetic field to the finger. 5- bone grafting and internal fixation of the distal phalanx. 64 Bone absorption that occurs with multiple myeloma is associated with which of the following? 1- Increased RANKL/osteoprotegerin ratio 4- Decreased TGF beta receptor 3- Decreased parathyroid-related hormone (PTrH) 2- Decreased levels of macrophage colony stimulating factor 5- Suppression of 1,25 dihydroxy vitamin D 65 Figures 65a through 65c are the radiograph and MRI scans of a 28-year-old man who reports recurrent right leg pain after undergoing an L4-5 microdiskectomy 2 years ago. Examination reveals weakness in the right ankle dorsiflexors (3/5) and decreased light touch sensation in the first web space of the right foot. Three 3 months of nonsurgical management has failed to provide relief. What is the most appropriate surgical treatment? 1- Microdiskectomy 2- Percutaneous diskectomy 3- Anterior diskectomy and fusion 4- Laminectomy and in situ fusion 5- Laminectomy and interbody fusion 66 With the forearm in full supination, the superficial radial nerve exits the muscular fascia to become subcutaneous along the 1- midportion of the supinator. 2- volar side of the brachioradialis. 3- dorsal side of the brachioradialis. 4- radial side of the flexor carpi radialis. 5- ulnar side of the flexor carpi radialis.

2011 Orthopaedic In-Training Examination • 17

67 When does the medial epicondyle apophysis of the elbow close in a male? 1- 4 to 7 years 2- 7 to 10 years 3- 10 to 13 years 4- 13 to 16 years 5- 16 to 19 years 68 Video 68 demonstrates stress examination of which of the following ligaments? 1- Bifurcate 2- Calcaneofibular 3- Anterior talofibular 4- Anterior inferior tibiofibular 5- Talocalcaneal interosseous 69 Figure 69 is the radiograph of a 25-year-old man who sustained a shoulder injury 5 years ago involving the axillary and suprascapular nerves. He underwent attempted neurolysis and latissimus dorsi tendon transfer without improvement in shoulder function. He now has profound deltoid atrophy and no motor function. The biceps is functional and hand function is normal. He maintains excellent control of the scapula. His current symptoms include pain, subjective instability, and limited active motion of 30 degrees of forward elevation and 0 degrees of external rotation. Electromyography shows no evidence of motor activity in the deltoid, supraspinatus, or infraspinatus. Management should consist of 1- hemiarthroplasty. 2- shoulder arthrodesis. 3- reverse total shoulder arthroplasty. 4- conventional total shoulder arthroplasty. 5- advising the patient that no further surgery would benefit him. 70 Quinolone antibiotics, such as ciprofloxacin, exert their effect through the inhibition of which bacterial enzyme? 1- DNA gyrase 2- DNA synthase 3- RNA methylase 4- RNA polymerase 5- Beta-lactamase

American Academy of Orthopaedic Surgeons • 18

71 After fracture of the articular surface, free devascularized segments of cartilage and bone are created. What is the best way to provide nutrition to surviving chondrocytes? 1- Hyperbaric oxygen therapy 2- Joint range of motion 3- Static positioning of the articulation 4- Slight distraction of the articular surface 5- Immediate weight bearing through opposing articular surface 72 Figures 72a through 72d are the radiographs and MRI scans of a 12-year-old boy with knee pain for 3 months. An intra-articular open biopsy specimen taken after arthroscopic drilling of an osteochondral defect is seen in Figure 72e. What is the best surgical option for local control and function of this contaminated joint and bone lesion? 1- Rotationplasty 2- Curettage and cementation 3- Above-the-knee amputation 4- Intercalary resection with allograft reconstruction 5- Distal femoral resection and endoprosthetic reconstruction 73 Which of the following is the most reliable anatomic landmark from which to base implant height when performing prosthetic hemiarthroplasty to manage a comminuted fracture of the proximal humerus? 1- Midportion of the coracoacromial ligament 2- Anterior insertion of the deltoid on the humerus 3- Proximal limit of the humeral fracture medially 4- Proximal limit of the intact intertubercular groove 5- Proximal border of the pectoralis major humeral insertion 74 Figure 74 shows the fracture of a 4-year-old boy with an avulsed nail bed. Treatment should consist of 1- closed reduction and pinning. 2- buddy taping to the fourth digit. 3- application of an aluminoform splint. 4- application of a distal interphalangeal joint extension splint. 5- irrigation, nail bed repair, and reduction. 75 When examining a patient with a crush injury to the lower leg, sensory and motor deficits are identified only in the distribution of the superficial peroneal nerve. In which compartment are the intramuscular pressures most likely to be elevated?

2011 Orthopaedic In-Training Examination • 19

1- Anterior 2- Lateral 3- Deep posterior 4- Superficial posterior 5- Plantar compartment of the foot 76 During collision sports, most cervical fractures and dislocations are due to which of the following? 1- A freak accident 2- Rotational injuries 3- Excessive lateral bending 4- Axial loading mechanisms 5- Flexion-distraction type mechanisms 77 Patients with sickle cell anemia are at increased risk for musculoskeletal infections caused by which of the following organisms? 1- Salmonella 2- Bordetella 3- Pseudomonas 4- Gram-negative bacilli 5- Pasteurella multicocida 78 The atraumatic lesion shown in Figures 78a through 78d (radiographs and intraoperative photos) is most commonly associated with which of the following? 1- Diabetes 2- Alcohol abuse 3- Hemoglobinopathies 4- Corticosteroid use 5- Long-standing tobacco use 79 What is the most significant risk factor for the development of a surgical site infection in a patient with rheumatoid arthritis of the foot? 1- Presence of rheumatoid nodules 2- Multiple joint involvement of arthropathy 3- History of previous surgical site infection 4- Failure to discontinue the use of methotrexate preoperatively 5- Failure to discontinue the use of anti-tumor necrosis factor medications preoperatively

American Academy of Orthopaedic Surgeons • 20

80 A patient sustained a posterior fracture-dislocation of the hip, and satisfactory reduction is achieved using closed reduction maneuvers. A postreduction CT scan reveals the presence of a concentric reduction, and a posterior wall fracture is identified. What is the best criteria for establishing joint stability and the need for surgical fixation? 1- Roof arc measurements 2- Posterior wall fragment size 3- Dynamic fluoroscopic stress test "push-pull" AP views 4- Dynamic fluoroscopic stress test in the iliac oblique view 5- Dynamic fluoroscopic stress test in the obturator oblique view 81 Which of the following figures represents the correct method to evaluate Southwick's slip angle in cases of slipped capital femoral epiphysis? 1- Figure 81a 2- Figure 81b 3- Figure 81c 4- Figure 81d 5- Figure 81e 82 Rehabilitation to address glenohumeral internal rotation deficit in the overhead athlete focuses on which of the following? 1- Strengthening the subscapularis 2- Strengthening the infraspinatus 3- Stretching the anterior capsule 4- Stretching the inferior capsule 5- Stretching the posterior capsule 83 Of the following possible risk factors, what is the most significant one for a community-acquired methicillin-resistant Staphylococcus aureus hand infection? 1- Inmate 2- Homeless 3- IV drug use 4- Diabetes mellitus 5- Human immunodeficiency virus 84 What is the most commonly reported complication with use of proximal humeral locking plates?

2011 Orthopaedic In-Training Examination • 21

1- Nonunion 2- Impingement 3- Screw penetration 4- Varus migration of the head 5- Osteonecrosis of the head 85 Orthopaedic surgeons performing history and examinations on patients receive the lowest patient satisfaction scores concerning what part of the interaction? 1- Not examining the patient 2- Not dealing with pain issues 3- Not spending enough time with the patient 4- Not getting a good history of the problem 5- Failure to offer empathetic responses 86 During cyclical loading of tendons and ligaments, the decrease in peak loads over time with the same amount of elongation is referred to as which of the following? 1- Creep 2- Wolff's law 3- Hueter-Volkmann law 4- Load relaxation 5- Elastic elongation 87 A 75-year-old woman underwent reverse shoulder arthroplasty 1 year ago for rotator cuff arthropathy with pseudoparalysis. While initially pleased with the results, by 6 months she reported increasing pain and now has severe pain and is unable to lift her arm from the side. A radiograph from her 6-month visit is seen in Figure 87a and a current radiograph is seen in Figure 87b. Neurologic examination is normal. Inflammatory laboratory studies and aspiration reveal no evidence of infection. What is the most likely reason for failure of the reverse prosthesis? 1- Deltoid dysfunction 2- Rotator cuff deficiency 3- Noncompliance with activity restrictions 4- Humeral component retroversion of 10 degrees 5- Superior placement of the baseplate and glenosphere 88 Which of the following diagnoses has the best 5-year survival if no metastatic disease is found at the time of diagnosis? 1- Ewing's sarcoma

American Academy of Orthopaedic Surgeons • 22

2- Parosteal osteosarcoma 3- Periosteal osteosarcoma 4- High-grade surface osteosarcoma 5- Secondary osteosarcoma from radiation therapy 89 Figures 89a and 89b are the parasagittal and axial CT scans of a 42-year-old patient seen after a motor vehicle collison. What anatomic structure is identified by the asterik in Figure 89b? 1- C6 facet 2- C7 facet 3- C6 pars 4- C6 pedicle 5- C7 pedicle 90 A 3-year-old girl with an L4 level myelomeningocele is ambulatory with a walker and ankle-foot orthoses. Radiographs reveal bilateral dislocated hips. On the right side, the acetabulum is quite shallow; however, on the left side it is well formed. What is the most appropriate treatment for both hips? 1- Continued nonsurgical management 2- Open reduction on the left side and closed reduction on the right side 3- Bilateral open reduction with external oblique transfer to the greater trochanter on the right side only 4- Bilateral open reduction of both hips with bilateral transfer of the iliopsoas to the greater trochanter 5- Bilateral open reduction of both hips with iliopsoas transfers to the greater trochanter bilaterally and right pelvic osteotomy 91 During distal locking of an antegrade femoral nail, the lateral fluoroscopic radiograph seen in Figure 91 is obtained. Before placing the interlocking screws, what maneuver should be performed? 1- Flexion of the hip and knee 2- Extension of the hip and knee 3- Adduction of the leg relative to the C-arm 4- Abduction of the leg relative to the C-arm 5- External rotation of the leg relative to the C-arm 92 While revising the acetabulum, a screw is placed in the posterior superior quadrant exiting the cortex. What neurovascular structure is at risk? 1- Sciatic nerve 2- Obturator nerve

2011 Orthopaedic In-Training Examination • 23

3- Femoral artery 4- External iliac vein 5- Superior gluteal vein 93 The meniscal condition seen in the arthroscopic Video 93 is best described as a 1- stable tear. 2- parrot beak-type tear. 3- horizontal cleavage tear. 4- bucket-handle type vertical tear. 5- discoid-variant medial meniscus. 94 During the deceleration phase of pitching, which of the following is the principal decelerator and is susceptible to tensile failure due to eccentric loading? 1- Deltoid 2- Brachialis 3- Rotator cuff 4- Pectoral major 5- Latissimus dorsi 95 A 35-year-old man is seen 6 months following a severe ankle sprain with persistent lateral ankle pain. Examination reveals a negative anterior drawer test and no ankle effusion. Lateral and mortise radiographs of the ankle are seen in Figures 95a and 95b, respectively. What treatment option is indicated for the problem illustrated by Video 95c? 1- Physical therapy program with peroneal strengthening and proprioceptive exercises 2- Application of a lace-up ankle brace 3- Lateral ankle ligament reconstruction 4- Peroneal tendon repair 5- Peroneal retinacular repair with a groove-deepening fibular osteotomy 96 Which of the following is considered an advantage of the deltopectoral approach compared with a transdeltoid ("deltoid splitting") approach when managing a proximal humerus fracture with plate fixation technique? 1- Less risk to the axillary nerve 2- Less disruption to remaining osseous blood supply 3- Less necessity for partial detachment of the deltoid insertion 4- Enhanced access to a greater tuberosity fracture 5- Exposure directly overlies the application site of the proximal portion of the plate

American Academy of Orthopaedic Surgeons • 24

97 A child with a nursemaids' elbow postures the arm in what position? 1- Full elbow extension and forearm pronation 2- Full elbow extension and forearm supination 3- Slightly flexed elbow and forearm pronation 4- Elbow flexion of at least 130 degrees and forearm pronation 5- Elbow flexion of between 100 and 130 degrees and forearm supination 98 A chromosome translocation t(X;18) that results in the fusion of the SYT gene and either the SSX1 or SSX2 gene is associated with 1- osteosarcoma. 2- rhabdomyosarcoma. 3- synovial sarcoma. 4- Ewing cell sarcoma. 5- histiocytosis X. 99 The fourth wrist extensor compartment is anatomically unique because of the presence of 1- bony side walls that contain the tendons. 2- more than one tendon within the compartment. 3- a separate subsheath that subdivides the compartment. 4- an acute change of direction of the tendon at the exit point. 5- a grossly visible sensory nerve running through the compartment. 100 Figures 100a through 100d are the radiographs and MRI scans of a 15-year-old girl with right hip pain for 2 months. A biopsy specimen is seen in Figure 100e. What is the most likely diagnosis? 1- Chondroblastoma 2- Giant cell tumor 3- Septic arthritis 4- Rheumatoid arthritis 5- Eosinophilic granuloma 101 On a standing lateral radiograph of the spine in which the edge of the film represents a true vertical, a normal C7 plumb line should intersect which of the following anatomic landmarks? 1- Center of the C7 vertebral body and the posterior-superior corner of S1 2- Center of the C7 vertebral body and the anterior-superior corner of S1

2011 Orthopaedic In-Training Examination • 25

3- Anterior border of the C7 vertebral body and the anterior-superior corner of S1 4- Anterior border of the C7 vertebral body and the posterior-superior corner of S1 5- Posterior border of the C7 vertebral body and the posterior-superior corner of S1 102 The best outcome following reverse total shoulder arthroplasty has been reported for what preoperative etiology? 1- Fracture sequelae 2- Rheumatoid arthritis 3- Rotator cuff tear arthropathy 4- Failed hemiarthroplasty 5- Conversion of proximal humerus resection 103 A 35-year-old man sustains an isolated, high-energy right pilon fracture. The brake travel time returns to normal (presumed ability to drive) how many weeks after the patient begins weight bearing? 1- 0 (when weight bearing begins) 2- 2 3- 4 4- 6 5- 8 104 In the stress-strain curve, the irreversible change in length after removing the load during the plastic range is called 1- elastic limit. 2- proportional limit. 3- breaking point. 4- ultimate strength. 5- plastic deformation. 105 What is the most common complication associated with the procedure seen in Figures 105a and 105b? 1- Nonunion 2- Patella baja 3- Arterial injury 4- Fracture of the lateral cortex 5- Compartment syndrome and peroneal nerve injury

American Academy of Orthopaedic Surgeons • 26

106 A 135-lb, 11-year-old boy sustained a closed comminuted midshaft femur fracture. What is the best instrumentation option? 1- Bridge plating 2- Flexible titanium nails 3- Closed reduction and spica cast 4- Solid retrograde intramedullary rod 5- Solid antegrade piriformis fossa entry intramedullary rod 107 Protraction of the scapula between the cocking and acceleration phases of throwing is mediated by what nerve? 1- Long thoracic 2- Thoracodorsal 3- Dorsal scapular 4- Spinal accessory 5- Medial pectoral 108 What is the most common anatomic variant in the relationship of the short external rotators and the sciatic nerve at the hip joint? 1- It passes, unsplit, through the piriformis muscle. 2- It travels between the piriformis and capsule, exiting posteriorly above the superior gemellus. 3- It travels between the capsule and the short external rotators exiting below the superior gemellus. 4- It splits above the piriformis with one branch passing through the muscle and the other below it. 5- It splits by the piriformis muscle so that a portion of the nerve is anterior and a portion is posterior. 109 The principles of tension band fixation are most applicable to which of the following fractures? 1- Comminuted patella 2- Comminuted olecranon 3- Non-comminuted tibial tubercle 4- Non-comminuted subtrochanteric femoral 5- Non-comminuted mid-femoral with vascular compromise requiring plate fixation medially 110 With respect to the following, in what manner have computer-assisted navigation systems proved advantageous? 1- Improved fracture healing times for humeral shaft fractures treated with medullary nails

2011 Orthopaedic In-Training Examination • 27

2- Improved long-term patient outcomes after percutaneous iliosacral screw fixation for pelvic ring disruptions 3- Reduced overall surgical time for medullary nailing of femoral shaft fractures 4- Reduced bladder perforation after percutaneous posterior column fixation for acetabular fractures 5- Reduced radiation exposure for surgeons performing freehand interlocking of medullary nails 111 Which of the following describes the phenomenon that occurs when a material is subjected to numerous loading cycles and then fails at a stress level lower than the ultimate stress for that material? 1- Creep 2- Fatigue 3- Toughness 4- Viscoelasticity 5- Plastic deformation 112 Figure 112 shows the radiographs of a 14-year-old boy with a several-month history of worsening wrist pain. Examination reveals tenderness in the mid-dorsal wrist. What is the most appropriate management? 1- Observation 2- Partial wrist fusion 3- Bone stimulator application 4- Temporary scaphotrapeziotrapezoidal pinning 5- Radial shortening osteotomy with plate fixation 113 A 23-year-old man has a 6-month history of worsening pain over the tip of his index finger. The pain is intermittent and exacerbated by cold or direct palpation. A radiograph was obtained (Figure 113a) and biopsy specimens are shown in Figures 113b (low power) and 113c (high power). What diagnosis is most consistent with these findings? 1- Melanoma 2- Enchondroma 3- Glomus tumor 4- Giant cell tumor 5- Osteomyelitis 114 Olecranon fracture fragment excision and triceps advancement is best indicated in which of the following scenarios? 1- A 30-year-old woman with a closed comminuted fracture involving more than 50% of the joint surface

American Academy of Orthopaedic Surgeons • 28

2- A 30-year-old woman with an open olecranon fracture which is proximal to the "bare spot" 3- A 55-year-old woman with an oblique olecranon fracture through the coronoid process 4- An 85-year-old man with a comminuted fracture involving less than 50% of the joint surface 5- An 85-year-old man with an oblique fracture through the coronoid process 115 A 57-year-old woman sustains a low-energy fracture of the distal radius. In evaluating for metabolic disorders, what is the most likely serum abnormality? 1- Elevated PTH 2- Decreased CTX 3- Decreased NTX 4- Decreased alkaline phosphatase 5- Low 25-hydroxy cholecalciferol (25 OH vitamin D) 116 Figure 116 is the standing lateral radiograph of a 67-year-old woman with a 4-week history of heel pain after a recent increase in walking activity. Pain occurs with every step and does not improve after a period of walking. The gastrocsoleus complex is tight and she reports pain when the sides of her heel are squeezed. What is the next best step in management? 1- CT scan 2- MRI scan 3- Bisphosphonates 4- Heel cord stretching 5- Corticosteroid injection 117 Figures 117a and 117b are the radiographs of an 8-year-old boy who holds his head at a slight tilt. Examination reveals decreased rotation and lateral flexion of the cervical spine. He is neurologically normal. In addition to obtaining flexion and extension views of the spine, what further management is necessary prior to making recommendations regarding sports participation? 1- Renal and cardiac evaluation 2- Lengthening of the sternocleidomastoid muscle 3- Surgical stabilization of the cervical spine 4- Three-dimensional CT evaluation of the cervical spine 5- Provide reassurance to the parents that no adverse long-term results should be expected 118 A 20-year-old man sustains the injury seen in Figures 118a and 118b in a fall from a bicycle. What is the most appropriate management? 1- Kenny-Howard splint 2- Figure of 8 brace for 6 weeks

2011 Orthopaedic In-Training Examination • 29

3- Figure of 8 brace for 3 weeks, followed by early motion 4- A sling for 3 to 6 weeks with early motion 5- Open reduction and internal fixation 119 In the evaluation of a patient, which of the following is a nonorganic sign of low back pain? 1- Proportional verbal or physical reaction 2- Positive straight-leg raise at 30 degrees 3- Back symptoms with axial loading 4- Hyperactive reflexes with patient distraction 5- Dermatomal sensory loss in the lower extremities 120 The outcome 1 year after rotator cuff-related surgery in patients with workers' compensation demonstrates which of the following? 1- No change in disability status between preoperative and postoperative 2- Better outcomes than non-workers' compensation patients 3- A lower level of satisfaction as compared to non-workers' compensation patients and unchanged function over their preoperative status 4- A lower level of satisfaction when compared with non-workers' compensation patients but improved function over their preoperative status 5- An equivalent level of satisfaction when compared with non-workers' compensation patients and improved function over their preoperative status 121 The progressive permanent deformation of materials in response to a constant force applied over an extended period of time is called 1- creep. 2- ductility. 3- anisotropy. 4- fatigue failure. 5- viscoelasticity. 122 Figures 122a and 122b are the radiographs of a 65-year-old man with a bump on his tibia for 3 years. A lateral radiograph from a year ago is seen in Figure 122c. MRI scans are seen in Figures 122d and 122e. Biopsy specimens are seen in Figures 122f and 122g. What is the most likely diagnosis? 1- Adamantinoma 2- Periosteal osteosarcoma 3- Small cell osteosarcoma 4- High-grade surface osteosarcoma

American Academy of Orthopaedic Surgeons • 30

5- Ossifying fibroma/osteofibrous dysplasia 123 What factor differentiates a Pipkin I from a Pipkin II femoral head fracture? 1- Degree of comminution 2- Direction of the hip dislocation 3- Location of the femoral head fracture 4- Presence of an associated femoral neck fracture 5- Size of the femoral head fracture fragment 124 The Immediate Post-Concussion Assessment and Cognitive Testing battery (IMPACT) has proven to be a valuable tool in determining the return to play criteria for injured athletes. Testing is performed via a computer-based protocol that evaluates attention, working memory, and which of the following? 1- Processing speed 2- Physical fitness 3- Retrograde amnesia 4- Hand-eye coordination 5- Loss of consciousness 125 A 12-month-old boy undergoes surgical release and full-thickness skin grafting for the congenital condition shown in Figures 125a and 125b. What is the most common postoperative complication that could negatively affect long-term function? 1- Web creep 2- Nail deformity 3- Digital nerve injury 4- Flexor tendon adhesions 5- Infection of the graft harvest site 126 A 60-year-old woman underwent total knee arthroplasty. Her twin brother underwent the same procedure last year. What gender-related differences could be expected? 1- Lower outcome scores in women 2- Improved range of motion in women 3- Improved implant survivorship in women 4- No difference as long as gender-specific implants are used 5- Higher incidence of postoperative patellofemoral pain in women

2011 Orthopaedic In-Training Examination • 31

127 A 12-year-old boy has a slipped capital femoral epiphysis. He can bear weight with assistive devices. What is the likelihood of development of osteonecrosis after in situ fixation? 1- Less than 10% 2- 10% to 35% 3- 36% to 65% 4- 66% to 90% 5- Greater than 90% 128 Figure 128 shows the clinical photograph of a patient performing a key pinch grip (pulp to pulp). What is the most likely explanation for the problem noted in the patient's left hand? 1- Ulnar nerve injury in the hand 2- Radial nerve injury at the elbow 3- Median nerve injury at the elbow 4- Median nerve injury at the wrist 5- Anterior interosseous nerve injury in the mid-forearm 129 A patient undergoing rehabilitation after anterior cruciate ligament reconstruction experiences postoperative arthrofibrosis. What clinical finding is most expected to be present? 1- Patella alta 2- Large knee effusion 3- Increased posterior drawer 4- Decreased patellar translation 5- Resolution of the extensor lag by postoperative day one 130 Figures 130a through 130c are the radiographs of a college baseball player obtained 7 days after sustaining an injury. He is tender to palpation over the carpal tunnel and denies any numbness or tingling in his fingers. What is the next most appropriate step in management? 1- CT scan of the wrist 2- MRI scan of the carpal tunnel 3- Complete blood count with differential 4- Referral to a neurologist for nerve conduction velocity studies 5- Hand therapy consultation for pain reduction and strengthening exercises 131 The proton energy produced by a 3.O Tesla machine is how many times greater than a 1.5 Tesla machine?

American Academy of Orthopaedic Surgeons • 32

1- 2 2- 4 3- 6 4- 9 5- 12 132 A 45-year-old man sustained a right sacroiliac joint dislocation and a symphysis rupture in a motorcycle accident. He was initially hypotensive and tachycardic and was given fluid resuscitation. Which of the following findings is the best indicator that he is fully resuscitated and ready for definitive skeletal stabilization? 1- Hemoglobin is normal 2- Heart rate has decreased 3- Blood pressure is normal 4- Base deficit is 0-3 mmol/L 5- Urine output is greater than 30 mL/h 133 A 16-year-old girl, admitted for management of scoliosis, tells the physician she does not want to do the preoperative pregnancy test because she has been having a sexual liaison with her school teacher and is worried she might be pregnant. She asks the physician to keep the information confidential. What is the physician's obligation? 1- Cancel the pregnancy test 2- Contact the police immediately 3- Notify child protective services and the parents 4- Notify the parents and let them decide whether the authorities are to be involved 5- Insist on the pregnancy test, and if it is positive, call pediatrics and cancel surgery 134 Figure 134 is the AP radiograph of a 28-year-old woman with progressively worsening pain along the lateral fifth metatarsal head over the last 2 years. She has pain in wide-toe box shoes. There are no calluses and the fifth metatarsophalangeal joint is stable. Which of the following is the most appropriate procedure? 1- Lateral eminence resection 4- Medial translation distal osteotomy 2- Proximal crescentic metatarsal osteotomy 3- Transverse plane metatarsal shaft osteotomy 5- Metatarsal shaft osteotomy in a plane midway between transverse and sagittal planes

2011 Orthopaedic In-Training Examination • 33

135 Figure 135 is the MRI scan of a 50-year-old mason with posterior elbow pain following a fall 2 days ago at work. Examination reveals moderate swelling at the posterior aspect of the elbow and he is unable to extend the arm against gravity. Management should consist of which of the following? 1- Surgery 2- Aspiration 3- Physical therapy 4- Cortisone injection 5- Cast immobilization 136 Which of the following scenarios involving treatment options for specific pediatric elbow fractures has the highest rate of developing a nonunion or fibrous union? 1- A Gartland type I supracondylar humerus fracture treated with a long-arm cast for 3 weeks 2- A Gartland type III supracondylar humerus fracture treated with closed reduction and percutaneous pinning 3- A 1-cm displaced lateral condyle fracture treated with open reduction and pin fixation 4- A 1-cm displaced medial epicondyle fracture treated with a long-arm cast for 3 weeks 5- A flexion-type supracondylar humerus fracture treated with open reduction and internal fixation 137 Figures 137a and 137b are the postoperative radiographs of a 64-year-old woman with a subtrochanteric fracture of the femur. She underwent antegrade femoral nailing employing a cephalomedullary device. Which of the following would minimize the risk of anterior perforation by the nail in the distal femur? 1- More anterior nail entry site 2- More posterior nail entry site 3- Nail of lesser radius of the curvature 4- Nail of greater radius of the curvature 5- Piriformis rather than trochanteric entry site 138 Methicillin-resistant Staphylococcus aureus creates an altered penicillin-binding protein that has a low affinity for beta-lactam antibiotics. The gene that codes for this protein is called 1- env. 2- src. 3- mecA. 4- Sox9. 5- MDR (multidrug resistance).

American Academy of Orthopaedic Surgeons • 34

139 Figures 139a and 139b are the radiographs of a 20-year-old man. What cell is most commonly abnormal with this condition? 1- Platelet 2- Osteoblast 3- Osteoclast 4- Osteocyte 5- Megakaryocyte 140 Internal impingement of the shoulder may be best described as contact between the 1- lesser tuberosity and the coracoid. 2- supraspinatus and the coracoacromial arch. 3- long head of the biceps and the coracoacromial ligament. 4- undersurface of the teres minor and the posterior glenoid labrum. 5- posterosuperior glenoid labrum and the greater tuberosity. 141 A comminuted olecranon fracture is best treated with which of the following constructs? 1- Bridge plating 2- Compression plating 3- A 7.3-mm lag screw alone 4- A 7.3-mm lag screw and tension band 5- Kirschner wire and tension band construct 142 The structure depicted ( * ) in the AP pelvic radiograph in Figure 142 is comprised of which of the following? 1- Ischial spine 2- Interspinous notch 3- Iliopectineal eminence 4- Quadrilateral surface and cotyloid fossa 5- Radiographic superimposition of the iliopectineal eminence and ischial spine

2011 Orthopaedic In-Training Examination • 35

143 Fanconi anemia is associated with what congenital deformity? 1- Brachydactyly 2- Preaxial polydactyly 3- Postaxial polydactyly 4- Ulnar longitudinal deficiency 5- Radial longitudinal deficiency 144 Core decompression of the distal radius for the treatment of Kienböck's disease is thought to work through which of the following mechanisms? 1- Unload the lunate fossa 2- Increase force distribution 3- Decrease distal radius stiffness 4- Decrease excessive intraosseous pressure 5- Incite local vascular healing response 145 Figures 145a through 145c are the radiographs and biopsy specimen of a 24-year-old woman who felt a painful pop in her finger when lifting a book. What is the best management of this lesion? 1- Chemotherapy 2- Ray amputation 3- Below-elbow amputation 4- Curettage and bone grafting 5- Wide resection and bone grafting 146 Figures 146a and 146b are the postoperative radiographs of an 83-year-old man with osteoarthritis who underwent a routine total knee arthroplasty. What is the most predictable solution to this problem? 1- Casting in extension for 6 weeks 2- Unlocked, hinged knee brace for 6 weeks 3- Revision with polyethylene exchange 4- Lateral collateral ligament reconstruction 5- Revision to a constrained condylar or hinged knee 147 A 36-year-old woman sustains an oblique, closed fracture of the humeral shaft 11 cms proximal to the lateral epicondyle with associated radial nerve palsy. The fracture is treated closed with the nerve injury treated expectantly. Which of the following is the first muscle expected to demonstrate evidence of reinnervation?

American Academy of Orthopaedic Surgeons • 36

1- Supinator 2- Brachioradialis 2- Extensor pollicis longus 3- Extensor indicis proprius 5- Extensor carpi radialis brevis 148 A patient underwent fixation of an acetabular fracture as seen in Figures 148a through 148c. Percutaneous screws were inserted under fluoroscopic guidance. Intraoperative views confirm desired intrapelvic positioning of the inferior screw. The fluoroscopic view seen in Figure 148d is obtained by which of the following methods? 1- Iliac oblique - inlet 2- Iliac oblique - outlet 3- Oburator oblique - inlet 4- Obturator oblique - outlet 5- AP beam orientation and neutral pelvic rotation 149 Figure 149 shows the clinical photograph of a 4-month-old child who has a deformity of the left hand and forearm. The thumb is absent. The right hand and forearm are normal. Laboratory studies show a normal CBC with platelets. What is the most appropriate surgical procedure? 1- Centralization of the ulna only 2- Pollicization of the index finger only 3- Pollicization of the index finger and centralization of the ulna 4- Rotational osteotomy and lengthening of the ulna 5- Vascularized transfer of the fibula to the forearm 150 A 70-year-old man hits his head in a fall. He has upper extremity weakness and dysesthesias. Examination reveals profound bilateral upper extremity weakness with 4/5 quadriceps and 5/5 anterior tibialis and gastrocsoleus strength. This finding is most consistent with which of the following syndromes? 1- Brown-Sequard 2- Central cord 3- Anterior cord 4- Posterior cord 5- Cervical myelopathic 151 What is the single most important predictive factor for successful healing of a Syme ankle disarticulation?

2011 Orthopaedic In-Training Examination • 37

1- Serum albumin level greater than 2.5 2- Ischemic index level greater than 0.45 3- Transcutaneous O2 levels of greater than 30 4- Total lymphocyte count of greater than 1,000 5- Posterior tibial artery Doppler signal 152 Which of the following nerves may travel within 1 mm of the inferior shoulder capsule and glenoid rim and is at risk during suture passage at the posterior inferior glenoid for shoulder stabilization procedures? 1- Suprascapular 2- Medial pectoral 3- Spinal accessory 4- Lower subscapular 5- Posterior branch of the axillary 153 Figures 153a and 153b are the radiographs of a 57-year-old woman who has had multiple joint complaints, back pain, and weakness for the past several years. Based on the shoulder and spine radiographs and a complaint of dark urine, a urine study is obtained. What is the most likely finding of the urine study? 1- High protein 2- Elevated uric acid level 3- Elevated homogentisic acid level 4- Calcium oxalate crystals 5- Maltese cross sign on polarized light of urine 154 A 19-year-old football player sustains a direct blow to the medial side of the tibia while being tackled. MRI reveals an isolated lateral collateral ligament tear of the knee. Examination would be expected to reveal which of the following? 1- Increased tibial external rotation at 30 degrees knee flexion 2- Increased varus laxity at 0 degrees 3- Increased varus laxity at 0 degrees and 30 degrees knee flexion 4- Increased varus laxity at 30 degrees knee flexion 5- Increased varus laxity at 0 degrees and increased tibial external rotation at 30 degrees knee flexion 155 Which of the following best describes the fracture seen in Figures 155a and 155b? 1- T-shaped 2- Anterior column

American Academy of Orthopaedic Surgeons • 38

3- Posterior column 4- Associated both column 5- Anterior with posterior hemitransverse 156 Which of the following displaced pediatric forearm fractures is the best candidate for intramedullary fixation with elastic nail(s)? 1- Salter Harris II distal radius fracture 2- Salter Harris IV proximal radius fracture 3- Salter Harris I distal radius and ulna fractures 4- Radius and ulna fractures at the mid diaphysis 5- Radius and ulna fractures at the distal metaphysis 157 Under existing Workers' Compensation laws, when is a patient able to choose a physician of their choice? 1- Never according to Federal Workers' Compensation laws 2- Always according to Federal Workers' Compensation laws 3- Never according to State Workers' Compensation laws 4- Always according to State Workers' Compensation laws 5- According to their individual State Workers' Compensation laws 158 A 35-year-old man has a 1-year history of plantar heel pain and lesser toe swelling. Examination reveals tenderness at the origin of the plantar fascia and there is fusiform swelling of all four of his lesser toes. There is no foot deformity. Which of the following physical findings is also most likely present? 1- Facial rash 2- Pitted fingernails 3- Palmar fascia contractures 4- First carpometacarpal arthrosis 5- Ulnar deviation of the fingers 159 Mendelian traits are those that follow specific patterns of inheritance controlled by which of the following? 1- Point mutations 2- Single gene pair 3- Autosomal chromosomes 4- Phenotypic expression 5- Genotypic transcription

2011 Orthopaedic In-Training Examination • 39

160 Hypophosphatasia is caused by decreased activity of which of the following gene products? 1- PTHrP 2- Vitamin D receptor 3- Alkaline phosphatase 4- Carbonic anhydrase II 5- Renal 25(OH) vitamin D 1 alpha-hydroxylase 161 A patient has a closed fracture of the tibia and fibula as seen in Figures 161a and 161b. Surgical treatment is elected. What technique modification would best ensure acceptable reduction with intramedullary nailing? 1- A more medial nail entry site 2- A more distal and anterior nail entry site 3- A coronally oriented blocking screw within the anterior portion of the proximal fragment 4- Nailing in a position of increased knee flexion 5- Temporary application of a unicortical plate 162 A premature infant is diagnosed with septic arthritis of the hip. The most critical next step is to evaluate for 1- child abuse. 2- necrotizing enterocolitis. 3- septic arthritis of another joint. 4- an underlying autoimmune disorder. 5- an underlying inflammatory disorder. 163 Cells thought to represent a chondrocyte stem cell population have been found in what zone of articular cartilage? 1- Deep 2- Tidemark 3- Superficial 4- Transitional 5- Calcified cartilage 164 Figures 164a and 164b are the radiographs of a 29-year-old man who experienced a tonic clonic seizure and awakened with severe left shoulder pain and loss of range of motion. He is treated with physical therapy for 6 weeks and, while the pain has subsided, he has loss of range of motion. His skin and

American Academy of Orthopaedic Surgeons • 40

neurovascular examinations are normal. Range of motion is limited to 60 degrees forward elevation, 0 degrees external rotation, and internal rotation to the midaxillary line. Passive and active range of motion are identical. What is the most appropriate management? 1- Labral repair 2- Total shoulder arthroplasty 3- Transfer of the subscapularis with the lesser tuberosity into the lesion 4- Transfer of the infraspinatus with the posterior greater tuberosity into the lesion 5- Continued physical therapy and counseling about the natural history of adhesive capsulitis 165 During placement of an iliosacral screw into the body of S1, it is important to obtain what pelvic radiographic view to help avoid injury to the S1 nerve root? 1- AP 2- Lateral 3- Inlet 4- Outlet 5- Obturator oblique 166 Figure 166 is the clinical photograph of a 38-year-old woman who sustained an injury to her right dominant hand in a manufacturing plant. Treatment should include 1- revision amputation of all digits because of the avulsion nature of the injury. 2- revision amputation because amputations through zone II should not be reimplanted. 3- replantation of all digits with forearm exposure for tendon reattachment. 4- replantation of the selected digit(s) based on anatomic site match (index to index, long to long, etc). 5- replantation of the selected digit(s) based on best intact parts proximally and distally, even if mismatched. 167 Persistent groin pain following total hip arthroplasty 6 months ago in a 58-year-old man (Figure 167) is most likely caused by which of the following? 1- Aseptic loosening 2- Iliopsoas impingement 3- Abductor insufficiency 4- Formation of a pseudotumor 5- Abnormal femoral version 168 The AAOS Clinical Guideline on Preventing PE in Arthroplasty Patients differed from the American College of Chest Physicians Guidelines by including which of the following prevention modalities?

2011 Orthopaedic In-Training Examination • 41

1- Clopidogrel 2- Aspirin 3- Heparin 4- Warfarin 5- Plasmaphoresis 169 Figure 169a is the lateral radiograph of a 19-year-old female gymnast with low back and leg pain. Examination reveals exacerbation of back pain with extension. She has a normal motor examination but diminished light touch sensation in an L5 distribution. What measurement shown in Figure 169b has been correlated with this disease? 1- Slip angle 2- Sacral slope 3- Pelvic tilt 4- Pelvic incidence 5- Spinopelvic angle 170 What nerve is most frequently injured during a two-incision distal biceps repair? 1- Ulnar 2- Median 3- Anterior interosseous 4- Posterior interosseous 5- Lateral antebrachial cutaneous 171 What gene product is most likely associated with multiple osteoblastic lesions due to metastatic prostate cancer? 1- BMP 4 2- Matrilin 3 3- Collagen Ib 4- Endothelin 1 5- ACVR1 receptor 172 Which of the following methods minimizes risk of fixation cut-out in an unstable 4-part intertrochanteric femur fracture fixed with a compression hip screw? 1- Use of a supplemental anti-rotation screw 2- Use of an extra large screw and deep thread design 3- Use of a Dimon-Hughston medialization to allow load sharing 4- Screw tip placement with a tip-apex distance of less than 25 mm

American Academy of Orthopaedic Surgeons • 42

5- Screw tip placement in the inferior hemisphere of the femoral head 173 Figures 173a and 173b are the clinical photograph and radiograph of a 9-year-old girl with a growth on her great toe for the past 4 months. It is painless but has been slowly enlarging. After appropriate preoperative workup, treatment should consist of 1- amputation of the toe. 2- amputation of the distal phalanx. 3- amputation followed by chemotherapy. 4- surgical excision of the lesion. 5- radiation therapy followed by amputation of the toe. 174 A patient allowed no more than touch-down weight bearing on the right lower extremity could use what type of assistive device support? 1- Two crutches 2- A single crutch in the left hand 3- A single crutch in the right hand 4- A cane in the left hand and a single crutch in the left hand 5- A cane in the right hand and a single crutch in the right hand 175 Cardiomyopathy is one of the dose-limiting toxicities of which of the following chemotherapy drugs? 1- Cisplatin 2- Doxorubicin 3- Iphosphamide 4- Methotrexate 5- VP-16 176 Figure 176 is the radiograph of a 50-year-old man with achiness in his left groin for 3 years. He states that he had a severe muscle injury when he slipped on the ice 4 years ago. What is the most likely diagnosis? 1- Synovial sarcoma 2- Myositis ossificans 3- Metastatic carcinoma 4- Foreign body reaction 5- Extraskeletal chondrosarcoma

2011 Orthopaedic In-Training Examination • 43

177 A 52-year-old man has difficulty walking 3 months after undergoing successful transmetatarsal amputation for ischemic gangrene. Observation of his gait reveals a shortened stance phase and abrupt abbreviated toe off on the surgical side. Which of the following devices would most likely improve his gait? 1- Solid ankle ankle-foot orthosis 2- Hinged ankle ankle-foot orthosis 3- Three-quarter length steel shank shoe modification 4- Full-length steel shank shoe modification 5- Full-length steel shank and rocker sole shoe modification 178 One year after undergoing medial compression screw fixation of a talar neck fracture with medial comminution, a 55-year-old man has a painful limp. Eversion is limited and radiographs reveal a healed fracture. What is the next most appropriate step in treatment? 1- Lateral column lengthening 2- Medial calcaneal sliding osteotomy 3- Crescenteric osteotomy of the talus 4- Opening medial wedge osteotomy of the talus 5- Closing lateral wedge osteotomy of the talus 179 Figures 179a and 179b are the MRI scans of a patient with shoulder pain and weakness. What muscle and nerve is most likely to be affected? 1- Deltoid and axillary 2- Teres minor and axillary 3- Supraspinatus and suprascapular 4- Infraspinatus and suprascapular 5- Subscapularis and lower subscapular 180 A hip arthroplasty performed through the anterior approach uses the plane between what nerves? 1- Superior gluteal and femoral 2- Inferior and superior gluteal 3- Obturator and superior gluteal 4- Obturator and lateral femoral cutaneous 5- Lateral femoral cutaneous and superior gluteal

American Academy of Orthopaedic Surgeons • 44

181 Which anatomic factor best differentiates an APC-II from an APC-III pelvic fracture pattern? 1- Amount of pubic symphysis diastasis 2- Disruption of the sacrotuberous ligament 3- Disruption of the anterior sacroiliac ligament 4- Disruption of the posterior sacroiliac ligament 5- Associated "crescent" fracture of the posterior iliac wing 182 Moderate or severe pain at 6 months after musculoskeletal injury can be as high as 38% depending on the type and extent of injury. Which of the following factors is a significant independent predictor of moderate or severe pain after musculoskeletal injury? 1- Male gender 2- Female gender 3- Age of younger than 65 years 4- High school graduate 5- History of pain interfering with activities 183 Fibrodysplasia ossificans progressiva, characterized by massive formation of spontaneous heterotopic bone, is caused by an altered signal transduction of which of the following proteins? 1- BMP-2 2- BMP-4 3- BMP-5 4- BMP-6 5- BMP-7 184 Figures 184a through 184c are the radiograph and MRI scans of an 18-year-old male soccer player who reports groin pain after several weeks of training. It worsens with prolonged practice and running activities and resolves with rest. What is the most appropriate management? 1- Physical therapy program 2- Percutaneous screw fixation 3- Core decompression with bone grafting 4- Hip arthroscopy for CAM-type femoroacetabular impingement 5- Relative rest to include cessation of impact activities for 6 weeks 185 Figures 185a and 185b are the radiographs of a patient who slipped on the stairs. CT scans do not reveal any associated bony lesions and confirm a dislocation in the medial two articulations. What is the preferred management that will optimize return of function?

2011 Orthopaedic In-Training Examination • 45

1- Closed reduction and casting 2- Closed reduction and percutaneous pinning 3- Open reduction and arthrodesis of the medial two tarsometatarsal joints 4- Open reduction and screw fixation across the medial two tarsometatarsal joints 5- Open reduction, excision of intra-articular loose bodies, and transarticular pinning 186 Minimally invasive total hip arthroplasty using a Watson-Jones approach as compared with a traditional transgluteal (anterolateral) approach results in which of the following? 1- Decreased blood loss 2- Decreased surgical time 3- Similar early gait kinematics 4- Increased risk of posterior dislocation 5- Increased risk of heterotopic bone formation 187 Figure 187 is the AP radiograph of a 3-month-old boy with thigh-fold asymmetry. The child was born Caesarian and is otherwise normal. Examination reveals that the Barlow and Ortolani tests are negative. He has 80 degrees of abduction in flexion on the left side and 60 degrees on the right side. Management should consist of 1- open reduction of the hip. 2- a follow-up radiograph in 3 months. 3- an arthrogram and closed reduction of the hip. 4- application of a Pavlik harness with ultrasound and clinical evaluation in 2 weeks. 5- application of a Pavlik harness with ultrasound and clinical evaluation in 8 weeks. 188 The femoral insertion of the medial patellofemoral ligament is closest to what letter in Figure 188? 1- A 2- B 3- C 4- D 5- E 189 Fixation of a displaced two-part pertrochanteric hip fracture is performed with a compression screw and side plate (dynamic hip screw). In contrast with a right hip fracture, a left hip fracture is more inclined to what form of malreduction? 1- Varus 2- Valgus

American Academy of Orthopaedic Surgeons • 46

3- Axial shortening 4- Proximal fragment flexion 5- Proximal fragment extension 190 A 67-year-old woman with painful rheumatoid arthritis presents for metacarpophalangeal joint arthroplasties of all four fingers. What is the expected functional outcome 1 year after surgery? 1- Increased flexion but no improvement of the ulnar drift 2- Decreased extensor lag and improvement of the ulnar drift 3- Decreased total range of motion but improvement of the ulnar drift 4- Improvement in total range of motion but increased extensor lag 5- No change in total range of motion or extensor lag 191 Figures 191a and 191b are the radiographs of a 66-year-old man who reports progressively worsening knee pain after undergoing total knee arthroplasty 3 years ago. He originally did well, but beginning 8 months after surgery began to have pain. Laboratory values reveal a C-reactive protein of 0.2 mg/dL (normal 0.0-0.6 mg/dL) and an erythrocyte sedimentation rate of 2 mm/h (normal 0-15 mm/h). What is the most likely cause of the pain? 1- Infection 2- Aseptic loosening 3- Synovial entrapment 4- Mid-flexion instability 5- Malpositioning of the components 192 What is the most likely long-term consequence of the untreated pediatric elbow injury seen in Figure 192? 1- Weakness of thumb adduction 2- Weakness of the thumb interphalangeal flexion and index distal interphalangeal flexion 3- Weakness of the wrist and thumb flexion 4- Weakness of the wrist and digit extension 5- Weakness of grasp and flexion of the little and ring fingers 193 In treating the fracture seen in Figure 193 with intramedullary nailing and using a blocking screw to assist with alignment during nail placement, what is the best position for a blocking screw? 1- Posterior to the nail in the distal fragment 2- Posterior to the nail in the proximal fragment 3- Anterior to the nail in the distal fragment 4- Anterior to the nail in the proximal fragment

2011 Orthopaedic In-Training Examination • 47

5- Anterior to the nail in the proximal fragment and posterior to the nail in the distal fragment 194 Teriparatide (Forteo) is contraindicated in patients with which of the following medical conditions? 1- Hypocalcemia 2- Renal failure 3- Osteoarthritis 4- Paget's disease 5- Rheumatoid arthritis 195 A morbidly obese, diabetic 50-year-old woman underwent knee arthroplasty and did well initially. Three weeks after surgery she has a sudden gush of fluid from the knee. She is seen in the emergency department and placed on antibiotics. Two days later clear fluid is still draining from her incision. Her C-reactive protein and erythrocyte sedimentation rate are normal. The knee is not red, hot, or painful. Range of motion is 0 to 120 degrees. What would be the next most appropriate step in management? 1- Arthroscopic debridement 2- Ten days of oral antibiotics 3- Six weeks of IV antibiotics 4- Irrigation and debridement with possible polyethylene exchange 5- Placement in a knee immobilizer and follow up in 3 days for reevaluation 196 Figures 196a through 196f show the radiographs, bone scans, and CT scans of a 57-year-old man with diabetes who has pain in his right leg. His history includes a vascularized right fibular bone graft to his left hip. Laboratory studies reveal a normal erythrocyte sedimentation rate and C-reactive protein. What is the most likely diagnosis? 1- Ewing's sarcoma 2- Stress fracture 3- Osteoid osteoma 4- Chronic osteomyelitis 5- Parosteal osteosarcoma 197 Which of the following is part of the "time-out" protocol recommended by the Joint Commission on Accreditation of Healthcare Organizations prior to any surgical or invasive procedure? 1- The circulating nurse should confirm the type and location of surgery with the patient's informed consent. 2- The patient should repeat the name of the procedure that is being performed. 3- The surgeon should initiate the time-out immediately after making the skin incision.

American Academy of Orthopaedic Surgeons • 48

4- The surgeon, anesthesiologist, and nurse should perform the time-out before entering the operating room. 5- The time-out should be performed at the end of surgery. 198 An 11-year-old girl sustains the fracture seen in Figures 198a and 198b. What are the maximal acceptable limits following closed reduction for this fracture? 1- Less than 10 degrees of angulation and less than 2 mm of displacement 2- Less than 10 degrees of angulation and less than 5 mm of displacement 3- Less than 30 degrees of angulation and less than 10% displacement 4- Less than 30 degrees of angulation and less than 30% displacement 5- Less than 60 degrees of angulation and less than 30% displacement 199 A patient has a subtrochanteric femoral fracture in which both the greater and lesser trochanters remain in continuity with the proximal fragment. What is the resultant deformity of the proximal fragment? 1- Abduction, flexion, internal rotation 2- Abduction, extension, internal rotation 3- Abduction, flexion, neutral rotation 4- Abduction, flexion, external rotation 5- Adduction, extension, external rotation 200 Which of the following is predictive of a poor outcome following latissimus dorsi transfer? 1- Deltoid tear 2- Teres minor tear 3- Subscapularis tear 4- Supraspinatus tear 5- Long head of the biceps tear 201 Figures 201a and 201b are the radiographs of a 50-year-old woman with progressive knee pain. An MRI scan shows a degenerative meniscal tear. Her BMI is 38. Based on AAOS guidelines, what treatment is recommended (Grade A level of evidence)? 1- Unispacer 2- Lateral heel wedge 3- Knee arthroscopy and debridement 4- Weight loss and low-impact exercise 5- Glucosamine and chondroitin sulfate

2011 Orthopaedic In-Training Examination • 49

202 A 72-year-old man has thigh discomfort. A radiograph reveals a mid-diaphyseal solitary femoral lesion with mineralization. He has a history of prostatic carcinoma diagnosed 10 years ago. What is the next most appropriate step in management? 1- Biopsy 2- Intramedullary stabilization 3- Intramedullary stabilization with cement augmentation 4- Presurgical embolization and intramedullary stabilization 5- Local irradiation followed by intramedullary stabilization 203 A 13-1/2-year-old boy of average height, whose skeletal age matches his chronologic age, is predicted to have a 3.4-cm limb-length inequality at skeletal maturity. The difference is primarily tibial. Which of the following is considered the best treatment to equalize limb length? 1- Immediate tibial epiphysiodesis 2- Immediate femoral epiphysiodesis 3- Immediate combined femoral and tibial epiphysiodesis 4- Tibial epiphysiodesis at age 15 years 5- Combined femoral and tibial epiphysiodesis at age 15 years 204 In designing a posterior cruciate ligament (PCL) reconstruction rehabilitation program, which of the following activities generates the least amount of stress within the PCL graft? 1- Isokinetic knee flexion 2- Non-weight-bearing resisted knee flexion 3- Non-weight-bearing resisted knee extension (60 to 0 degrees) 4- Non-weight-bearing knee extension from full flexion to about 70 degrees 5- Weight-bearing exercises with depths of greater than 70 degrees of knee flexion 205 Examination of a 20-year-old man who was involved in a motor vehicle accident reveals that he is obtunded with hypotension and tachycardia. The pelvic radiographs seen in Figures 205a through 205c reveal which of the following findings? 1- Osteoid osteoma 2- Osteochondritis dissecans 3- Os acetabuli marginalis superior 4- Anterior wall acetabular fracture 5- Anterior column acetabular fracture

American Academy of Orthopaedic Surgeons • 50

206 In the preparation and administration of platelet-rich plasma, what is the role of calcium chloride? 1- Reduced viscosity 2- Reduced immunogenicity 3- Initiation of platelet activation 4- Increased angiogenesis 5- Increased expression of prostaglandin E2 207 Figures 207a and 207b are the radiographs of a 14-year-old girl with osteogenesis imperfecta. She has no back pain and has had no radiographic progression during the past year. She is neurologically intact. The osteogenesis imperfecta has been treated with infusions of pamidronate. What is the next most appropriate step in management? 1- Continued observation 2- Arthrodesis from T4 through L4 3- Arthrodesis from T4 to the sacrum 4- Arthrodesis from T10 through L3 5- Application of a custom molded thoracolumbosacral orthosis 208 Figure 208 is the radiograph of a 73-year-old woman with a history of hip dysplasia who underwent revision total hip arthroplasty and awoke from general anesthesia unable to dorsiflex her ankle on the surgical side. Management should consist of 1- an MRI scan of the lumbar spine. 2- immediate hip extension and knee flexion. 3- serial laboratory evaluation including hemoglobin and coagulation factors. 4- a return to the operating room for immediate acetabular screw removal. 5- a return to the operating room for immediate sciatic nerve exploration. 209 A 55-year-old man with neck pain is referred for emergent consultation because of concerns about spinal cord compression on the MRI scans shown in Figures 209a and 209b. He has no upper or lower extremity neurologic symptoms and has normal gait and balance. Examination reveals normal motor and sensory function but hyperreflexia in bilateral triceps and quadriceps. What is the most appropriate management? 1- Observation 2- Anterior decompression and fusion C3-7 3- Posterior laminaplasty C3-7 4- Posterior laminectomy and fusion C3-C7 5- Combined anterior and posterior decompression and fusion C3-7

2011 Orthopaedic In-Training Examination • 51

210 Tumorigenesis, or the development of a tumor with the ability to metastasize, is a multi-step process characterized as which of the following? 1- Genomic stability 2- Sustained angiogenesis 3- Programmed cell death (apoptosis) 4- Protease activity down-regulation 5- Limited replicative potential (telomerase activity) 211 What soft-tissue structure is most likely to be deficient in a patient with the congenital anomaly shown in Figure 211? 1- Iliotibial band 2- Ligament teres 3- Anterior cruciate ligament 4- Lateral collateral ligament 5- Femoral acetabular ligament 212 Buttress plating technique is best used for what type of fracture? 1- Simple partial articular 2- Simple complete articular 3- Comminuted nonarticular 4- Comminuted partial articular 5- Comminuted complete articular 213 The deformity seen in Figures 213a and 213b is most consistent with which of the following? 1- Distal biceps rupture 2- Acute lacertus strain 3- Rupture of the rotator cuff 4- Rupture of the brachioradialis 5- Rupture of the long head of the biceps brachii 214 An 11-year-old girl has the deformity seen in Figures 214a and 214b. Past surgical history is significant for a clubfoot corrective surgery at 6 months of age. What is the cause of this deformity? 1- Insufficiency of the plantar fascia 2- Unopposed pull of the anterior tibialis muscle 3- Congenital deformity of the talonavicular joint

American Academy of Orthopaedic Surgeons • 52

4- Overlengthening of the posterior tibial tendon 5- Excessive lengthening of the peroneus brevis tendon 215 A patient has recurrent instability with pivoting activities following anterior cruciate ligament reconstruction. Examination reveals a 2A Lachman, 1A anterior drawer, and grade 3 pivot shift. External rotation dial testing at 30 degrees and 90 degrees is negative. A radiograph (Figure 215b) and image (Figure 215a) from the subsequent arthroscopic examination reveals what likely error during the initial procedure? 1- Vertical graft placement 2- Unrecognized posterolateral corner instability 3- Staple fixation of the graft on the tibial side 4- Posterior graft placement with back wall blowout 5- Horizontal graft placement with an excessively horizontal tibial tunnel 216 Figure 216 is the AP radiograph of a 14-year-old girl with osteogenesis imperfecta. She has been on recurrent IV infusion treatment with biphosphonates since age 4. What is the explanation of the horizontal lines? 1- Growth arrest lines 2- Biphosphonate treatment 3- Indications of prior fractures 4- Result of her primary disease 5- Result of intermittent lead exposure 217 Which of the following is necessary during the informed consent process for use in clinical trials? 1- No informed consent is necessary if the two treatment options being evaluated are considered to have equal risk. 2- The process for obtaining informed consent for patients included in clinical trials is mandated by the Institutional Review Board (IRB). 3- The informed consent process only needs to detail the potential benefits of the trial procedure. 4- In obtaining the consent, only the experimental arm of the trial needs to be explained because it is being compared with a control that is more commonly accepted as a reasonable form of treatment. 5- The IRB cannot dictate the process of informed consent. 218 Locking plate fixation is employed to manage a displaced two-part proximal humerus surgical neck fracture in a 68-year-old woman. What method best ensures preservation of the reduction? 1- Deliberate varus reduction 2- Insertion of a laterally positioned intramedullary fibular strut graft

2011 Orthopaedic In-Training Examination • 53

3- Insertion of inferomedial locking ("calcar") screws within the humeral head 4- Insertion of nonlocking "pull" screws subsequent to locking screws within the humeral head 5- Inclusion of suture fixation within the osseous portions of the tuberosities 219 Which of the following is the post-hoc statistical correction that is performed when comparing multiple independent means in a series of pair-wise comparisons in order to determine where the differences occur? 1- Power analysis 2- Student's t-test 3- Yates correction 4- Bonferroni correction 5- Analysis of variance (ANOVA) 220 Based on the findings shown in Figures 220a through 220h, the images and Video 220i portrayed are most consistent with which of the following? 1- Bankart lesion 2- Rotator cuff tear 3- Posterior labral tear 4- Anterior labral periosteal sleeve avulsion (ALPSA) lesion 5- Humeral avulsion of the glenohumeral ligaments (HAGL) lesion 221 Which of the following statements is true about the use of a cane? 1- Use in the ipsilateral hand reduces the abductor angle. 2- Use in the contralateral hand may reduce joint reactive force of up to 50%. 3- Cane use reduces the body weight lever arm. 4- Cane use will prolong the life of total hip arthroplasty. 5- Cane use cannot change the joint reactive force, only improve balance. 222 A 55-year-old woman has a stage 2 acquired adult flatfoot deformity secondary to posterior tibial tendon insufficiency. With the patient seated and the hindfoot reduced to subtalar joint neutral by the examiner, there is residual deformity as shown in Figure 222. What component of the flatfoot reconstruction procedure will address this deformity? 1- Medial displacement calcaneal osteotomy 2- Medial cuneiform opening wedge (Cotton) osteotomy 3- Anterior calcaneal neck lengthening (Evans) osteotomy 4- Flexor digitorum longus tendon transfer to the navicular 5- Peroneus longus tendon repair and reefing

American Academy of Orthopaedic Surgeons • 54

223 Which of the following treatments has been shown to improve bone density, decrease fracture incidence, and improve function when administered to young children with Sillence type 3 osteogenesis imperfecta? 1- Etanercept 2- Chondroitin 3- Bisphosphonates 4- Collagen type A 5- 1,25-dihydroxy vitamin D 224 A minimally invasive plate osteosynthesis is seen in Figures 224a and 224b. What is the most important reason to open a skin portal distally during plate insertion? 1- Needed for proper insertion of the plate 2- To facilitate reduction of the fracture 3- To protect the anterior tibial artery and the deep peroneal nerve 4- To prevent entrapment of the tibialis anterior muscle and tendon 5- To decompress the anterior compartment to prevent compartment syndrome 225 An 18-year-old man who fell 36 hours ago sustained a C4-5 bilateral facet dislocation with a complete spinal cord injury (ASIA A). Prior to treatment, he and his family are adamant that high-dose methylprednisolone be given. What is the most appropriate response? 1- Contact the hospital's risk management team. 2- Ignore the request because of risks of treatment and limited reported benefits. 3- Conduct an open dialogue with the patient and family and explain the risks and benefits. 4- Administer steroids 30 mg/kg bolus at an infusion rate of 5.4 mg/kg/hour x 24 hours. 5- Administer steroids 30 mg/kg bolus at an infusion rate of 5.4 mg/kg/hour x 48 hours. 226 The U.S. Food and Drug Administration has recommended that use of elastomeric devices for the continuous intra-articular infusion of local anesthetics should be stopped primarily due to concern that 1- catheter tips were breaking unexpectedly. 2- the devices are being used in an off-label manner. 3- patients may be sensitized to the latex in the device. 4- an increased risk of surgical site infection has been observed. 5- local anesthetics are associated with chondrolysis of the operative joint. 227 Which of the following findings are typical of femoral fractures that occur in patients after long-term alendronate therapy?

2011 Orthopaedic In-Training Examination • 55

1- Low energy, spiral, atrophic cortices 2- Low energy, transverse, atrophic cortices 3- Low energy, transverse, hypertrophic cortices 4- High energy, transverse, hypertrophic cortices 5- High energy, transverse, normal cortices 228 Figures 228a and 228b are the AP radiographs of a 17-year-old boy with an elbow deformity. He has a range of motion from 0 to 110 degrees of flexion and is neurologically intact with no pain. If the deformity is treated surgically, which of the following procedures would help in achieving a cosmetically and functionally pleasing result? 1- Medialize the distal fragment 2- Lateralize the distal fragment 3- Close the capitellar epiphysis 4- Transpose the ulnar nerve anteriorly 5- Excise the growth arrest zone medially 229 A female patient walks with an "antalgic gait". What gait abnormality does the term antalgic describe? 1- Unsteady wide-based gait 2- Pelvic tilt at midstance 3- Shortened stance on the affected limb 4- Advancing leg lifted high to clear toes 5- Decreased quadriceps moment in midstance 230 During a lateral approach to the shoulder, the axillary nerve can be found at approximately what distance from the tip of the acromium? 1- 3 cm 2- 5 cm 3- 7 cm 4- 9 cm 5- 11 cm 231 A 77-year-old woman has significant right shoulder pain aggravated by activity and significant positional night pain that awakens her. She denies any recent trauma or any constitutional symptoms. Examination reveals marked atrophy of the infraspinatus but no deltoid atrophy. Active overhead elevation is to 160 degrees but with a painful arc. She has weakness of external rotation by the side, with an external rotation lag by the side but no external rotation lag in the 90-degree abducted position. An abdominal compression test is normal. Yergason's test is markedly positive. Nonsteroidal anti-

American Academy of Orthopaedic Surgeons • 56

inflammatory drugs have failed to provide relief. Physical therapy aggravates her pain. Radiographs are seen in Figures 231a and 231b and a representative MRI scan is seen in Figure 231c. What is the best treatment option at this time? 1- Total shoulder arthroplasty 2- Reverse total shoulder arthroplasty 3- Arthroscopic rotator cuff repair 4- Arthroscopic debridement and biceps tenotomy 5- Reconstruction of the coracoacromial arch with allograft 232 A 1-month-old girl has weakness of the right upper extremity that has been present since birth. In the work-up, which of the following findings would suggest a favorable outcome? 1- Ptosis 2- Twitch biceps activity 3- Periscapular muscle atrophy 4- Motor activity on electromyography 5- Meningoceles on magnetic resonance imaging 233 Figures 233a and 233b are the radiographs of a 60-year-old man who underwent surgical fixation to manage a combined mid-diaphyseal and proximal femoral fracture. At his 8-month follow-up visit, he now reports groin pain with attempted full weight-bearing ambulation. Treatment should now include 1- revision hip screw fixation with retention of the device. 2- revision fixation with endosteal plating and fibular strut allograft. 3- implant removal and valgus osteotomy with blade fixation. 4- implant removal and valgus osteotomy with cephalomedullary nailing. 5- implant removal and exchange nailing with a cephalomedullary device. 234 A deep, 5-cm soft-tissue mass is removed without prior radiographic evaluation, expecting it to be a lipoma. The mass was intracompartmental with good hemostasis. The final pathology report lists the diagnosis as a high-grade pleomorphic sarcoma. Treatment should include which of the following? 1- Amputation 2- Radiation therapy without more surgery 3- Chemotherapy without more surgery 4- Close observation for the next 2 years 5- Radical re-resection of the tumor bed and radiation therapy

2011 Orthopaedic In-Training Examination • 57

235 Figure 235 is the radiograph of a 1-year-old child. Which line best represents Hilgrenreiner's line? 1- A 2- B 3- C 4- D 5- E 236 Figure 236 is the MRI scan of a patient with pain and weakness in the leg. What muscle is most likely to be weak? 1- Gastrocnemius 2- Anterior tibialis 3- Posterior tibialis 4- Peroneus longus 5- Extensor hallucis longus 237 What is the most frequent complication encountered in the surgical treatment of the injury seen in Figures 237a through 237d? 1- Nonunion 2- Infection 3- Motion loss 4- Recurrent instability 5- Subsequent development of meniscal pathology 238 After undergoing a total hip arthroplasty, a 54-year-old woman reports that the operated leg feels longer and a therapist has recommended putting a shoe lift in the other shoe. Standing pelvis and supine pelvis radiographs are seen in Figures 238a and 238b. What is the cause of her limb-length discrepancy? 1- Abductor weakness 2- Adduction contracture 3- Fixed pelvic obliquity 4- Increased acetabular anteversion 5- Lengthening of the leg due to prosthesis position 239 A non-randomized prospective cohort study that compares the results of treatment provides what level of evidence?

American Academy of Orthopaedic Surgeons • 58

1- I 2- II 3- III 4- IV 5- V 240 Figure 240 is the radiograph of a 50-year-old woman with a 5-year history of a painful bunion that interferes with daily activity and most shoe wear. There is no pain with hallux metatarsophalangeal joint range of motion. The hallux can be passively overcorrected into varus. Which of the following is the most appropriate corrective surgical procedure? 1- Medial eminence resection 2- Distal first metatarsal osteotomy bunionectomy 3- Proximal first metatarsal osteotomy bunionectomy 4- Hallux proximal phalanx medial closing wedge osteotomy 5- First tarsometatarsal angulatory correction with arthrodesis 241 To maximize functional recovery following open reduction and internal fixation of a posterior wall acetabular fracture, rehabilitation 3 months after surgery should focus on 1- gait speed. 2- hamstring strengthening. 3- hip abductor strengthening. 4- hip range of motion. 5- knee and ankle range of motion. 242 Figure 242 is the oblique radiograph of a 42-year-old woman who had breast carcinoma 3 years ago and now has plantar heel pain. She has similar cortical thickening on her medial tibia. What is the most likely diagnosis? 1- Melorheostosis 2- Periosteal osteoma 3- Metastatic carcinoma 4- Healed stress fracture 5- Parosteal osteosarcoma 243 What type of comparison in a clinical trial excludes patients who were not compliant with the protocol guidelines? 1- Parametric 2- Non-parametric

2011 Orthopaedic In-Training Examination • 59

3- Per protocol 4- Intent to treat 5- Cross-over design 244 A 30-year-old recreational hockey player sustained a closed simple elbow dislocation 5 years ago, which was treated nonsurgically. He generally has a painless elbow with excellent range of motion. Activities such as pushing off from a chair cause him pain and clicking in the posterior aspect of his elbow. Examination reveals full range of motion, no neurovascular deficits, and excellent strength. A posterolateral rotatory pivot shift test is seen on Video 244. Nonsurgical management has been unsuccessful and the patient desires surgical treatment. Which of the following procedures is most likely to be successful? 1- Direct repair of the radial collateral ligament 2- Direct repair of the lateral ulnar collateral ligament 3- Reconstruction of the annular ligament with graft 4- Reconstruction of the radial collateral ligament with graft 5- Reconstruction of the lateral ulnar collateral ligament with graft 245 A patient undergoes retrograde femoral nailing as seen in Figures 245a and 245b. What structure is most at risk during the course of proximal interlocking in the sagittal plane? 1- Lateral femoral cutaneous nerve 2- Divisions of the superficial femoral artery 3- Divisions of the femoral nerve and deep femoral artery 4- Tendinous origin of the direct head of the rectus femoris 5- Tendinous origin of the indirect head of the rectus femoris 246 Figure 246 is the radiograph of a 64-year-old woman who has persistent pain and weakness after undergoing basilar thumb reconstruction with flexor carpi radialis tendon harvest and interposition. Examination reveals an unstable thumb metacarpal base and hyperextension of the metacarpophalangeal (MP) joint to more than 60 degrees. Treatment should now consist of 1- percutaneous fixation of the MP joint. 2- percutaneous fixation of the thumb metacarpal base. 3- basilar thumb ligament reconstruction. 4- basilar thumb ligament reconstruction and MP joint fusion. 5- basilar thumb ligament reconstruction and percutaneous fixation of the MP joint. 247 Figures 247a and 247b are the radiographs of a 60-year-old man with intermittent hallux pain while playing tennis for the last 6 weeks. There is no pain with range of motion except at maximum dorsiflexion. The grind test is negative. Which of the following is the most appropriate treatment?

American Academy of Orthopaedic Surgeons • 60

1- Cheilectomy 2- Morton's extension foot orthosis 3- Longitudinal arch support with metatarsal pad 4- First metatarsophalangeal arthrodesis 5- First metatarsophalangeal joint implant arthroplasty 248 The CT scan findings shown in Figure 248 are most consistent with which of the following Young and Burgess pelvic fracture classification patterns? 1- VS 2- APC-II 3- APC-III 4- LC-II 5- LC-III 249 A 15-year-old girl with acetabular dysplasia requires a pelvic osteotomy. The hip is concentric with a center edge angle of 5 degrees. Which of the following is the most appropriate type of osteotomy? 1- Chiari 2- Dega 3- Ganz 4- Salter 5- Pemberton 250 A patient underwent total knee arthroplasty 6 weeks ago. He has 70 degrees of flexion despite undergoing aggressive physical therapy. Preoperatively and intraoperatively, he had 110 degrees of motion as seen in Figure 250. Treatment should now consist of 1- open lysis of adhesions. 2- arthroscopic lysis of adhesions. 3- manipulation under anesthesia. 4- decreasing polyethylene thickness. 5- tibial and femoral component revision. 251 A 70-year-old man has had severe neck pain and upper and lower extremity pain for the past 2 years. He reports progressive gait instability with a number of falls. Examination reveals weakness in biceps, triceps, and hand intrinsics bilaterally. He has 3+ biceps, triceps, and quadriceps reflexes with 10 beats

2011 Orthopaedic In-Training Examination • 61

of ankle clonus. A radiograph and MRI scan are seen in Figures 251a and 251b. Which of the following is the strongest contraindication to posterior decompression? 1- Age 2- Severe neck pain 3- Cervical kyphosis 4- Duration of symptoms 5- Multiple-level disease 252 When performing unconstrained total shoulder arthroplasty, a humeral neck osteotomy performed with the correct version and inclination but with excessive bone removal will cause what complication? 1- Glenoid malposition 2- Rotator cuff injury 3- Brachial plexus injury 4- Humeral stem loosening 5- Overstuffing of the joint 253 Figures 253a and 253b are the MRI scans of an 18-year-old female gymnast with pain while tumbling. The pain is exacerbated by plantar flexion. She denies any inversion injuries. Rest, activity restriction, and physical therapy have not improved her symptoms. Treatment should now consist of 1- excision os trigonum. 2- anterior ankle debridement. 3- Achilles tendon debridement. 4- peroneal tendon debridement. 5- lateral ankle ligament reconstruction. 254 When considering implant options for managing a displaced subtrochanteric fracture of the proximal femur, what is the principal benefit of intramedullary nailing in contrast with plating? 1- Diminished injury to the abductors 2- Implant insertion facilitates reduction 3- More ideally manages preexisiting deformity 4- Biomechanic superiority and diminished risk of implant failure 5- Uniquely suited to indirect reduction and minimally invasive fixation 255 The anterior interosseous nerve innervates which of the following muscles?

American Academy of Orthopaedic Surgeons • 62

1- Flexor digitorum profundus to the index and middle fingers, flexor pollicis longus, pronator quadratus 2- Flexor digitorum profundus to the index, middle, ring, little fingers, flexor pollicis longus, pronator quadratus 3- Flexor digitorum profundus to the ring, little fingers, flexor pollicis longus, flexor pollicis brevis 4- Flexor digitorum profundus to the index, middle fingers, flexor pollicis longus, abductor pollicis brevis 5- Flexor digitorum profundus to the index, middle fingers, flexor pollicis longus, lumbricals to the index and middle fingers 256 Following a fall, a 67-year-old Medicare patient is seen for a new wrist deformity at the request of the primary care physician. The patient was seen by your partner 5 years ago for treatment of a compression spine fracture. After evaluation, documenting your findings, and sending communication to the requesting physician, the most appropriate category for your services is which of the following? 1- New patient (CPT code 99203); Medicare no longer recognizes consultation codes 2- Return patient (CPT code 99213); the patient is known to your practice 3- Consultation (CPT code 99243); the requirements for consultation are met 4- Postoperative visit (CPT code 99024); the patient is post spine fracture treatment 5- Second opinion confirmatory (CPT code 99273); you are the second opinion after that of the primary care physician 257 The National Osteoporosis Foundation recommends a daily calcium intake of how many milligrams for men and women older than age 50? 1- 250-500 mg 2- 750 mg 3- 1,200-1,500 mg 4- 2,000-3,000 mg 5- 5,000 mg 258 Where does the piriformis muscle originate? 1- Inner table of the iliac wing 2- Inner side of the obturator membrane 3- Outer table of the iliac wing 4- Outer side of the obturator membrane 5- Ventral surface of the sacrum 259 Which of the following best describes the technique used to obtain a false profile view of the hip?

2011 Orthopaedic In-Training Examination • 63

1- Standing AP position with the hip flexed 65 degrees 2- Standing position at an angle of 65 degrees between the pelvis and the film 3- Supine position with the hip abducted 65 degrees 4- Supine position with an oblique angle of 65 degrees between the pelvis and the film 5- Sitting position with an oblique angle of 65 degrees between the pelvis and the film 260 Figures 260a and 260b are the weight-bearing AP and lateral radiographs of a 61-year-old woman with progressively worsening pain at the first metatarsophalangeal joint and increasing angulation of the hallux over the last 3 years. Examination reveals significant pronation of the hallux. What is the most appropriate corrective surgical procedure? 1- Distal first metatarsal osteotomy 2- Distal first metatarsal osteotomy with lateral metatarsophalangeal joint soft-tissue release 3- Proximal first metatarsal osteotomy 4- Proximal first metatarsal osteotomy with lateral metatarsophalangeal joint soft-tissue release 5- First metatarsophalangeal arthrodesis 261 A 62-year-old right-hand dominant man who has a history of rheumatoid arthritis underwent a primary total elbow arthroplasty 1 year ago. He continues to report elbow pain since the initial surgery. He has both rest and night pain and denies any drainage from the wound although he does state that at times "my elbow feels hot." He denies any systemic symptoms. A plain radiograph is seen in Figure 261. His peripheral white blood cell count is 5,500/mm3 (normal 3,500-10,500/mm3) with 80% neutrophils. His C-reactive protein level is 15. Aspiration was performed and cultures grew proprionibacterium acnes. Treatment should now consist of 1- resection arthroplasty. 2- staged exchange arthroplasty. 3- single stage exchange arthroplasty. 4- arthroscopic irrigation and debridement. 5- open irrigation and debridement with bushing exchange. 262 The structure indicated by the arrows in the arthroscopic image of a hip (Figure 262) is important in what type of hip injuries? 1- Distraction 2- Hyperflexion 3- Hyperabduction 4- External rotation 5- Pipken-type femoral head fractures

American Academy of Orthopaedic Surgeons • 64

263 Which of the following factors has been associated with a greater risk of nonunion among patients with a type II odontoid fracture? 1- Diabetes 2- Male gender 3- Angulation of 5 degrees 4- Patient age of less than 65 years 5- Posterior displacement of 10 mm 264 Compared with standard infrapatellar nailing of a proximal-third tibia fracture, the semi-extended technique for intramedullary nailing is beneficial because it 1- does not violate the intra-articular space of the knee. 2- minimizes the apex anterior deformity at the fracture site. 3- reduces the risk of iatrogenic injury to the menisci. 4- reduces the risk of saphenous nerve injury. 5- increases the length of the nail that can safely be inserted. 265 According to the American Academy of Orthopaedic Surgeons clinical guidelines, which of the following has the strongest evidence to support its use in the nonsurgical treatment of carpal tunnel syndrome? 1- Acupuncture 2- Iontophoresis 3- Wrist splinting 4- Heat therapy 5- Massage therapy 266 During a posterior approach to the hip, Video 266 shows a stitch being placed in what structure? 1- Posterior capsule 2- Piriformis tendon 3- Gluteus medius tendon 4- Gluteus minimus tendon 5- Inferior gemellus tendon 267 Which of the following is most commonly associated with the use of calcium sulfate alone in the treatment of fracture nonunions? 1- 90% rate of healing of the nonunion

2011 Orthopaedic In-Training Examination • 65

2- Increased osteoinductivity at the nonunion site 3- Increased rate of infection at the surgical site 4- Increased rate of serous drainage at the surgical site 5- Increased likelihood of refracture after successful healing of the nonunion 268 Figure 268 is the radiograph of a 5-year-old boy who sustained a fracture of his right humerus. Initial treatment should consist of which of the following? 1- Immobilization 2- Biopsy of the cyst 3- Bone marrow injection 4- Flexible titanium intramedullary nails 5- Cortisone injections with or without bone graft 269 Which schematic seen in Figure 269 represents the ideal cross-sectional screw configuration within the femoral neck for treatment of a subcapital hip fracture? 1- a 2- b 3- c 4- d 5- e 270 Figure 270 is the CT scan of a 32-year-old woman who was in a motor vehicle collision. She has neck and right arm pain. Neurologic examination is normal except for right biceps weakness (4/5). She has no other injuries. Definitive treatment should include 1- a rigid cervical orthosis. 2- anterior C5-6 arthrodesis. 3- posterior C4-6 arthrodesis. 4- posterior C5-6 arthrodesis. 5- bed rest and high-dose methylprednisolone. 271 In designing an anterior cruciate ligament (ACL) reconstruction rehabilitation program, which of the following activities generates the least amount of stress within the ACL graft? 1- Straight leg raise 2- Resisted leg raises with 30 degrees of knee flexion 3- Slow speed (<60 degrees/sec) isokinetic knee flexion 4- Slow-speed (<60 degrees/sec) isokinetic knee extension 5- Non-weight-bearing resisted terminal knee extension (30 to 0 degrees)

American Academy of Orthopaedic Surgeons • 66

272 A disorder transmitted as an autosomal-dominant trait with genetic anticipation implies which of the following? 1- 25% of offspring, both males and females, will have the disorder and the ones who inherit the disease will be likely to have a less severe disease than the involved parent. 2- 50% of male and female offspring will have the disorder and the ones who inherit it will be likely to have a more severe disease than the involved parent. 3- 50% of male offspring will have the disorder and females who inherit the disease will have fewer tandem repeats than the involved parent. 4- 50% of female offspring will have the disorder and males who inherit the disease will have fewer tandem repeats than the involved parent. 5- The percentage of offspring who inherit the disorder increases with each subsequent generation. 273 Figure 273 are the radiographs of a 50-year-old man who sustained a wrist injury after a motorcycle accident. There is no distal radioulnar joint instability after radius fixation. Management should include fixation of the radius 1- alone. 2- and ulnar styloid excision. 3- and repair of the triangular fibrocartilage complex. 4- and percutaneous fixation of the ulnar styloid. 5- and open reduction and internal fixation of the ulnar styloid. 274 Figures 274a and 274b are the AP and lateral radiographs of a 12-year-old boy with medial knee pain and effusion after a twisting injury. Management should consist of 1- diagnostic arthroscopy and microfracture. 2- diagnostic arthroscopy and retrograde drilling. 3- diagnostic arthroscopy and osteochondral autograft transfer. 4- non-weight-bearing with crutches and a long leg cast for 8 weeks and close follow-up. 5- non-weight-bearing with crutches, continued knee range-of-motion exercises, and close follow-up. 275 Figure 275 is the MRI scan of a 75-year-old man who has had atraumatic right shoulder pain during overhead activity for the past month. Examination reveals 160 degrees of active and passive bilateral shoulder elevation. The right shoulder has pain with 90 degrees of forward flexion with full internal rotation. Waist level external rotation strength and abdominal compression tests are normal. Treatment should consist of which of the following? 1- Electromyography 2- Acromioplasty

2011 Orthopaedic In-Training Examination • 67

3- Physical therapy 4- SLAP repair 5- Rotator cuff repair